PEAT 3 Flashcards

1
Q

During pregnancy, which of the following exercises is CONTRAINDICATED?
A. Curl-ups
B. Bridging
C. Double leg lifts
D. Deep breathing with forced expiration

A

C. During pregnancy and postpartum, the stretched abdominal muscles are unable to stabilize the lower back as the legs are raised. Attempting to perform double leg lifts can overwork the abdominal muscles and cause damage to spinal joints.

How well did you know this?
1
Not at all
2
3
4
5
Perfectly
2
Q

A target heart rate is determined for individuals entering a training program in order to:
A. regulate exercise intensity.
B. estimate energy expenditure
C. control blood pressure at a specific heart rate.
D. ensure that participants exercise at maximal capacity.

A

A. Exercise intensity can be expressed as oxygen uptake during activity. Heart rate and oxygen uptake have a relatively linear relationship. Therefore, utilizing a target heart rate will ensure that the appropriate exercise intensity is being achieved. Estimation of energy expenditure requires measurement of oxygen consumption that is then calculated into calories metabolized during the activity. Blood pressure cannot be controlled at specific heart rates. Exercising at maximal capacity does not achieve aerobic training benefits and is unsafe for the majority of patient populations.

How well did you know this?
1
Not at all
2
3
4
5
Perfectly
3
Q

For a child with Duchenne muscular dystrophy, the MOST appropriate physical therapy goal is:
A. prevention of contractures and determination of the best method of mobility
B. preservation of strength and muscle tone.
C. inhibition of abnormal muscle tone and facilitation of normal movement and postural reactions
D. facilitation of normal movement and improvement of strength.

A

A. Goals of PT intervention for a child with Duchenne muscular dystrophy are to retard the development of contracture and muscle weakness, which could lead to functional limitations, and thus, disability. The physical therapist would also play a role in determining the appropriate use of assistive devices that could help maintain the child’s mobility such as wheelchairs, walkers and orthoses. Muscle tone changes and declines in strength cannot be prevented since they are results of the disease process.

How well did you know this?
1
Not at all
2
3
4
5
Perfectly
4
Q

Outcomes of a prenatal exercise program would NOT include
A. improved body mechanics.
B. application of relaxation techniques.
C. improved ligamentous flexibility.
D. strengthened pelvic-floor musculature

A

C During pregnancy, the ligaments soften due to hormonal influences, and allow some degree of separation between the joint surfaces. Additional stretching of the ligaments could result in joint instability or injury, and would not be a goal of treatment. The remaining options are all appropriate interventions.

How well did you know this?
1
Not at all
2
3
4
5
Perfectly
5
Q

A patient has limited ankle dorsiflexion following open reduction and internal fixation of the distal tibia. Radiographs reveal that the fracture is well healed. Treatment with passive mobilization should include:
A. posterior glide of the talus on the tibia.
B. lateral glide of the calcaneus on the tibia.
C. posterior glide of the tibia on the talus.
D. anterior glide of the talus on the tibia.

A

A
The trochlea of the talus is convex. Therefore, during dorsiflexion of the ankle, the talus moves posteriorly relative to the tibia. To facilitate the movement of dorsiflexion one would need to perform a posterior glide of the talus on the tibia.

How well did you know this?
1
Not at all
2
3
4
5
Perfectly
6
Q

A patient who has recently and successfully completed a 12-week program of Phase III cardiac rehabilitation will MOST likely demonstrate a decrease in:
A. carbon dioxide elimination in maximal work
B. cardiac output in maximal work.
C. stroke volume at a given level of submaximal work.
D. heart rate at a given level of submaximal work.

A

D
Aerobic conditioning that occurs during the 12 weeks of cardiac rehabilitation will result in a decrease in heart rate both at rest and with exercise. CO2 elimination and cardiac output would both increase with maximal work. The stroke volume would increase during submaximal work.

How well did you know this?
1
Not at all
2
3
4
5
Perfectly
7
Q

Redness on the inferior aspect of the patella upon removal of a patellar-tendon-bearing prosthesis indicates that the residual limb:
A. is not far enough into the prosthesis, and fewer socks should be worn
B. is not far enough into the prosthesis, and additional socks should be worn
C. has slipped too far into the prosthesis, and fewer socks should be worn.
D. has slipped too far into the prosthesis, and additional socks should be worn

A

D The residual limb is pistoning up and down because the socket diameter is too large or the suspension system is inadequate, resulting in skin friction. The addition of more socks will enhance the fit.

How well did you know this?
1
Not at all
2
3
4
5
Perfectly
8
Q

A physical therapist is evaluating a patient with an acute lumbar disc protrusion and a right lateral shift of the thoracic spine. Which of the following findings is the BEST indicator that the symptoms will respond positively to physical therapy intervention?
A. The pain is referred only into the buttock and not the thigh
B. The patient prefers standing and walking to sitting.
C. There is a decrease in lumbar lordosis
D. Repeated backward bending centralizes the pain

A

D The fact that backward bending is centralizing the pain is an indicator that the nucleus of the disc is being moved centrally, and the pressure is being taken off of the nerve root. Centralization of the pain is also a good indicator for physical therapy intervention.

How well did you know this?
1
Not at all
2
3
4
5
Perfectly
9
Q

A physical therapist evaluates a patient who has lateral epicondylitis. The patient reports a subjective pain rating of 8/10. The patient also reports pain and shows weakness with resisted wrist extension. The therapist decides to use ice massage as an intervention. The MOST appropriate length of time for the ice massage is:
A. 10 minutes.
B. until the area turns red, and the patient reports a burning sensation
C. until the patient reports that the area feels numb
D. no longer than 5 minutes.

A

C Ice massage is usually applied to control pain, edema, or inflammation. In this case, the ice would be used for pain relief and to reduce inflammation, if present. The dosage for ice massage is determined by the patient’s response, and is usually applied until the patient experiences analgesia or reported numbness over the area of the massage. Although 5 to 10 minutes may be a usual time for the response to occur, the dosage depends on the patient’s response, not on an exact time. The patient will usually feel the following, sequential sensations during the massage: cold, burning, aching and then finally numbness. The desired effect is numbness, not aching. The skin may turn white; however, the desired effect is numbness or pain reduction and not skin color.

How well did you know this?
1
Not at all
2
3
4
5
Perfectly
10
Q

To conduct an experimental study on pain in postsurgical orthopedic patients, a physical therapist randomly assigns patients into 2 groups. One group is treated with transcutaneous electrical nerve stimulation, heat, and exercise; the second receives heat and exercise only. In this experimental design, transcutaneous electrical nerve stimulation is the:
A. continuous variable.
B. dependent variable
C. discrete variable
D. independent variable.

A

D The independent variable can be thought of as the cause or treatment and the dependent variable can be thought of as the effect or response. In this case the TENS is the treatment or independent variable. Continuous and discrete variables are methods of quantifying variables

How well did you know this?
1
Not at all
2
3
4
5
Perfectly
11
Q

It is important to perform cool-down exercises immediately following a general aerobic exercise program PRIMARILY in order to prevent:
A. venous pooling.
B. cardiac arrhythmia.
C. decreased body temperature
D. muscle tightening.

A

A During aerobic exercise, there is vascular dilation that occurs in order to optimize blood flow and the venous system relies on muscular pumping in order to return blood back to the heart.
Without a cool-down period, there would be no muscular pumping and therefore blood would pool in the venous system. Cardiac arrhythmia may occur as well, but is less likely in someone without cardiac disease. Cessation of activity by itself would result in a gradual decrease in body temperature and specific exercises are not necessary. Muscle tightening may occur as a result of metabolites accumulating in the circulatory system, but this would occur as a result of the venous pooling

How well did you know this?
1
Not at all
2
3
4
5
Perfectly
12
Q

The intervention for a patient who has right sciatic pain caused by piriformis compression should
NOT include:
A. instruction in mild self-stretching in sitting with the right hip and knee flexed and pressure applied in the medial direction to the distal thigh with the left upper extremity.
B. contract-relax exercises to the hip external rotators performed with the patient sidelying on the left and the right hip and knee positioned in 90° of flexion
C. active resistive strengthening exercises to the piriformis with the patient prone and the knee flexed.
D. application of cold to the area of sciatic nerve irritability.

A

C The piriformis muscle functions as an external rotator of the hip, and it is thought that a tight piriformis muscle may compress the sciatic nerve causing pain. Passive internal rotation and resisted external rotation may be painful. Intervention would call for stretching of the piriformis muscle, not strengthening it. Modalities such as ice may also be helpful to decrease the inflammation.

How well did you know this?
1
Not at all
2
3
4
5
Perfectly
13
Q

A physical therapist is assigned the planning and implementation of physical therapy service for all members of a community. The therapist’s FIRST step should be to:
A. develop a brochure for distribution to the community.
B. organize a health fair to provide screening for the community.
C. evaluate existing services and community resources.
D. initiate contacts in the medical community to establish prescriptive relationships.

A

C The most appropriate first step (pre-planning) would be to evaluate existing services within the community and work out a budget for the project. Options A and B, although important, are things that would most likely be done after the practice is up and running. Option D may be done early on in the planning process, but would follow Option C.

How well did you know this?
1
Not at all
2
3
4
5
Perfectly
14
Q

A patient is referred to physical therapy reporting severe pain in the right hip and groin area, which increases during walking. The patient reports tenderness when the therapist palpates the area over the right greater trochanter. The MOST likely cause of the patient’s signs and symptoms is:
A. sacroiliac joint derangement.
B. a hip fracture.
C. a strain of the adductor longus muscle.
D. hip bursitis.

A

D Signs and symptoms of hip bursitis include the following: severe pain over the bursa area, with pain aggravated by active motion including activities such as walking. Signs and symptoms of a sacroiliac joint derangement include pain directly over the region of the joint and in the low back, in addition to pain with walking. Signs and symptoms of a hip fracture will include the following: severe pain in the groin area and tenderness occurs in the area anterior to the femoral neck. An adductor longus muscle strain would not cause tenderness over the greater trochanter

How well did you know this?
1
Not at all
2
3
4
5
Perfectly
15
Q

When working with neurological patients to ensure that physical therapy services are adequately documented, it is MOST important to record changes in:
A. muscle tone.
B. functional abilities.
C. cognitive status.
D. quality of movement.

A

B All documentation about PT services should readily translate the physical findings (impairments) into functional abilities/limitations.

How well did you know this?
1
Not at all
2
3
4
5
Perfectly
16
Q

A patient has right hemiparesis resulting from a traumatic brain injury. When assessing motor control in the right lower extremity with the patient standing, the physical therapist finds that the patient cannot extend the hip while flexing the knee or flex the hip while extending the knee. In which of the following functional activities will this problem be MOST apparent?
A. Shifting weight while standing
B. Walking sideways
C. Walking backward
D. Moving from a sitting position to a standing position

A

C Of the four options, backward walking is the only one that requires hip extension with concurrent knee flexion and hip flexion with knee extension

How well did you know this?
1
Not at all
2
3
4
5
Perfectly
17
Q

A physical therapist examining wrist-joint play finds restriction in the direction indicated by the arrow. To address the restriction, the therapist should include an intervention to increase which motion of the index finger (2nd digit)?
A. Flexion
B. Extension
C. Abduction
D. Rotation

A

A The therapist is shown performing a volar glide, which is the same, joint motion used for finger flexion. Limited motion in this direction indicates limited ability to perform finger flexion.

How well did you know this?
1
Not at all
2
3
4
5
Perfectly
18
Q

In which of the following conditions is a nerve conduction velocity test MOST appropriate?
A. Carpal tunnel syndrome
B. Cerebrovascular accident
C. Myotonia
D. Duchenne muscular dystrophy

A

A Nerve conduction velocity testing is most useful in the evaluation of peripheral nerve or lower motor neuron status. Therefore, since carpal tunnel syndrome is the only one of the conditions that directly involves a peripheral nerve; it would be the most appropriate choice. A cerebrovascular accident is an upper motor neuron disorder. Both myotonia and Duchenne muscular dystrophy are primary muscle disorders

How well did you know this?
1
Not at all
2
3
4
5
Perfectly
19
Q

A patient with leukemia has developed thrombocytopenia following a bone-marrow transplant.
Which of the following measures are indicative of the status of the thrombocytopenia?
A. Ta lymphocyte count
B. Red blood cell count
C. Platelet count
D. White blood cell count

A

C Thrombocytopenia is an acute or chronic decrease in the number of platelets in the circulation. The Ta lymphocyte count is used to assess immune status in patients with HIV or AIDS. The red blood cell count is utilized to assess for presence of anemia and the white blood cell count would be examined to determine presence of infection or degree of immunosuppression

How well did you know this?
1
Not at all
2
3
4
5
Perfectly
20
Q

For a patient with insulin-dependent diabetes who is completing a cardiovascular fitness program, what change in diabetic management is MOST likely to be instituted as fitness increases?
A. Switching to oral rather than injected medication
B. Decreasing caloric intake for 2 to 3 hours following exercise sessions
C. Decreasing the amount of insulin taken daily
D. Increasing the amount of insulin taken daily

A

C Exercise has been shown to increase sensitivity of the insulin receptors therefore leading to a decrease in the amount required. Administration of medication is dictated by tolerance and efficacy of the medication and would not necessarily be altered by exercise. In patients with moderate hyperglycemia, exercise can lead to hypoglycemia for periods of 24 to 48 hours after exercise, therefore increasing caloric intake, particularly carbohydrates, would be essential

How well did you know this?
1
Not at all
2
3
4
5
Perfectly
21
Q

An elderly patient presents with a sacral pressure ulcer measuring 6 in x6 in (15 cm x 15 cm). The wound has moderate serous fluid drainage and is loosely covered with necrotic and fibrotic tissue, although there are no indications of infection present. The BEST method of debridement is:
A. daily vigorous scrubbing of the wound.
B. wet-to-dry dressings with normal saline 2 times/day.
C. daily wet-to-dry dressings with 1:1 diluted povidone-iodine (Betadine).
D. whirlpool jet agitation 2 times/day.

A

B Wet-to-dry dressings are indicated for necrotic tissue needing debridement. The moderate amount of drainage would require more frequent (i.e., bid) dressing changes. Vigorous scrubbing of the wound could damage friable tissue. Betadine can be cytotoxic. Whirlpool would not be effective in removing necrotic tissue.

How well did you know this?
1
Not at all
2
3
4
5
Perfectly
22
Q

A patient has disuse atrophy of the anterior compartment muscles following cast removal after a fracture of the tibia. The patient has Poor (2/5) strength in the ankle dorsiflexors. Electrical stimulation is to be used to enhance dorsiflexor strength in conjunction with exercise. Initially, which of the following waveform characteristics are MOST appropriate to stimulate the muscles?
A. Monophasic pulse with a frequency of 100 pps and an on/off cycle ratio of 3:1
B. Biphasic pulse with a frequency of 30 ps and an on/off cycle ratio of 1:5
C. Direct current waveform with an on/off ratio of 1:1
D. Interferential waveform with a beat frequency of 1 pps

A

B The most efficient stimulus would be one that causes tetany in the muscles) without causing fatigue. The biphasic waveform at 30 ps best fits this stimulus. The 100 ps stimulus rate may cause fatigue, which would be undesirable. The interferential waveform at a frequency of 1 pps is too low and the direct current would not be appropriate because it would not cause tetany. In addition an on/off cycle of 1:5 would allow adequate recovery time for the muscles between successive contractions.

How well did you know this?
1
Not at all
2
3
4
5
Perfectly
23
Q

A patient is referred to physical therapy with a 20° knee flexion contracture following arthroscopic knee surgery 1 month ago. All of the following interventions are appropriate EXCEPT:
A. stretching of the semimembranosus muscle
B. strengthening of the vastus medialis.
C. inferior gliding of the patella.
D. mobility of the posterior capsule of the knee joint.

A

C A 20° knee flexion contracture means that the knee is unable to complete the last 20° of extension. This limitation of motion can be caused by several factors including tightness of the hamstrings (semimembranosus), restriction in the posterior capsule of the knee, and weakness of the quadriceps femoris muscles (vastus medialis). Restriction of patellar movement may also be a factor, however the proper glide technique to increase knee extension would be a superior (not inferior) glide of the patella.

How well did you know this?
1
Not at all
2
3
4
5
Perfectly
24
Q

During an evaluation, a patient lacks 10° of passive ankle dorsiflexion. The same degree of limitation is present whether the knee is flexed or extended. The muscle MOST likely contributing
to this restriction is the
A. gastrocnemius.
B. tibialis anterior.
C. plantaris.
D. soleus.

A

D The soles originates on the tibia and fibula and inserts onto the calcaneus, crossing only the ankle joint. Therefore, it will have an effect on the ankle whether the knee is flexed or extended. The gastrocnemius and plantaris both have origins on the femur and insert onto the calcaneus crossing both the knee and ankle joints. Therefore, they will be placed on stretch with the knee extended and will potentially limit ankle dorsiflexion to a greater extent with the knee extended. The tibialis anterior is a dorsiflexor of the ankle and while weakness in this muscle may limit active dorsiflexion, it would not affect passive dorsiflexion.

How well did you know this?
1
Not at all
2
3
4
5
Perfectly
25
Q

A patient sustained a severe brain stem injury 1 week ago and has demonstrated minimal change since the incident. The patient’s cognitive status is MOST likely to include:
A. a decreased level of arousal
B. a decreased level of intelligence
C. an increased level of agitation
D. an increased level of impulsivity

A

A A state of alertness to the internal and external environment must be maintained for motor or mental activity to occur. The brain stem reticular activating system brings about this state of general arousal. To proceed from a state of general arousal to one of “selective attention” requires the communication of information to and from the cortex, the thalamus, and the limbic system and its modulation over the brainstem and spinal pattern generators.

How well did you know this?
1
Not at all
2
3
4
5
Perfectly
26
Q

To maximize continuity of patient care, the physical therapist should begin discharge planning
when the:
A. physician refers the patient for discharge
B. therapist makes the discharge evaluation of the patient
C. discharge planner requests information pertinent to the patient’s discharge status.
D. therapist performs the initial evaluation of the patient.

A

D The Standards of Practice for Physical Therapy state that the physical therapist is responsible for establishing a plan of care for the patient based on the evaluation of the examination data and patient needs. The plan of care includes plans for discharge. The implication is that the long-term goals for the patient (discharge plans) are determined when the therapist first sees the patient. It is anticipated that modifications of the plan of care is to be expected depending on the response of the patient to the intervention.

How well did you know this?
1
Not at all
2
3
4
5
Perfectly
27
Q

A manual muscle test of a patient who sustained a gunshot wound just superior to the elbow joint reveals specific muscle weakness from a partial median nerve injury. The physical therapy intervention for this patient should include strengthening activities for wrist flexion, forearm:
A. pronation, finger flexion, and thumb adduction.
B. pronation, finger flexion, and thumb opposition.
C. supination, finger abduction, and thumb opposition
D. supination, finger flexion, and thumb extension.

A

B The median nerve innervates the following muscles in the forearm: (1) pronator teres and quadratus, (2) flexor digitorum superficialis, (3) flexor digitorum profundus (index and middle fingers), (4) thenar muscles (abductor pollicis brevis, opponens pollicis, flexor pollicis brevis).
Therefore, a lesion of the median nerve would affect those muscles and their accompanying actions: forearm pronation, finger flexion and thumb opposition. Thumb adduction is accomplished by the adductor pollicis (ulnar nerve). Finger abduction is performed by the dorsal interossei (ulnar nerve). Forearm supination is the action of the supinator (radial nerve) and biceps brachii (musculocutaneous nerve).

How well did you know this?
1
Not at all
2
3
4
5
Perfectly
28
Q

A measurable objective for a community education program on proper exercise techniques would be for participants to:
A. understand the importance of a sufficient warm-up period.
B. list 5 stretching techniques that can be used when warming up
C. overcome lower extremity problems and adhere to a regular exercise program.
D. appreciate the effects of increasing intensity of exercise on heart rate.

A

B
Objectives must be measurable and the specific behavior expected should be stated.
Understanding, overcoming, and appreciating elude tangible measurement, while listing five techniques is an activity that can be documented and is therefore measurable.

How well did you know this?
1
Not at all
2
3
4
5
Perfectly
29
Q

During manual muscle testing of the hip flexors in the sitting position, a patient exhibits lateral (external) rotation with abduction of the thigh as resistance is applied. The physical therapist should suspect muscle substitution by the:
A. sartorius.
B. tensor fascia late.
C. adductor longus.
D. semimembranosus.

A

A The sartorius flexes, externally rotates and abducts the hip joint. With resisted hip flexion, the sartorius will be recruited to perform all three actions giving the observed substitution pattern.
The tensor fascia late is a medial (internal) rotator and flexor of the hip, so substitution by it would involve medial (internal) rotation and abduction. The adductor longs would adduct the hip.
Substitution by the semimembranosus would cause hip extension.

How well did you know this?
1
Not at all
2
3
4
5
Perfectly
30
Q

A physical therapist is treating a patient who has lymphedema in the right lower extremity. The therapist decides to use mechanical, intermittent pneumatic compression as an intervention, using a sleeve that covers the foot, leg, and thigh. The therapist measures the patient’s blood pressure at 135/85 mm Hg. At the first treatment session, the MOST appropriate inflation pressure for the sleeve is:
A. 20 mm Hg.
B. 60 mm Hg.
C. 100 mm Hg.
D. 140 mm Hg.

A

B The recommended dosage for mechanical, intermittent pneumatic compression pressure is between 30 and 80 mm Hg, (higher for the lower extremities than upper extremities). It is also recommended that the pressure be below the diastolic pressure. Pressures below 30 mm Hg are not considered to be effective. Pressures above diastolic can be used with caution. For the first treatment, it would be most appropriate to use a pressure, which is considered effective, yet not so high as to cause potential problems for the patient. If the patient responds positively to the intervention, a higher pressure could be instituted.

How well did you know this?
1
Not at all
2
3
4
5
Perfectly
31
Q

A physical therapist receives a telephone call from a woman who identifies herself as a friend of one of the therapist’s patients. She wants to know how the patient is doing and if he will be able to go up and down the stairs, because she wants to take him home for a weekend visit. Which of the following actions is MOST appropriate for the therapist to take
A. Discuss the patient’s program and functional status with the caller.
B. Invite the caller to observe the patient’s next therapy session.
C. Refuse to discuss the patient, unless the patient’s permission is obtained
D. Refer the caller to the patient’s social worker.

A

C According to the Guide to Physical Therapist Practice, “Information relating to the physical therapist/patient relationship is confidential and may not be communicated to a third party not involved in that patient’s care without the prior written consent of the patient.” Option C is the only one that fits these criteria.

How well did you know this?
1
Not at all
2
3
4
5
Perfectly
32
Q

The parents of a 1 month-old child with myelomeningocele report that the child has been increasingly irritable and has changed sleeping patterns over the past 3 days. The child has also been vomiting. Which of the following actions is MOST appropriate for the therapist to take?
A. Refer for immediate medical follow-up.
B. Instruct the parents to monitor head circumference daily
C. Advise the parents to watch the child for worsening symptoms.
D. Take head circumference measurements to compare at subsequent visits

A

A Sixty percent of children with myelomeningocele develop hydrocephalus after surgical closure of their lesion. Early warning signs of hydrocephalus include irritability; changes in sleep patterns, and changes in appetite and weight. Eighty to ninety percent of children who acquire hydrocephalus will require a cerebrospinal fluid shunt. These children require ongoing follow-up by a physician. Given the seriousness of this child’s symptoms, the child should be immediately referred for medical follow-up.

How well did you know this?
1
Not at all
2
3
4
5
Perfectly
33
Q

A physical therapist is developing a gait training program for a patient following a total hip arthroplasty of the right hip done via an anterolateral approach 2 weeks ago. The proper instructions and rationale for crutch training utilizing 1 crutch include holding the crutch in the:
A. right hand to decrease activity in the right hip abductors.
B. right hand to facilitate activity in the right hip abductors.
C. left hand to decrease activity in the right hip abductors
D. left hand to facilitate activity in the right hip abductors

A

C
With an anterolateral approach, the gluteus medius is reflected or the trochanter is taken down. Post operatively, these structures need time to heal and will not be healed only 2 weeks after surgery. The abductors should not be aggressively strengthened with exercise.

How well did you know this?
1
Not at all
2
3
4
5
Perfectly
34
Q

A right-handed woman is being examined by a physical therapist for right-sided weakness in her upper and lower extremities. Her muscle tone is hypotonic and she is ataxic during walking. She has a history of hypertension and alcoholism. Passive range of motion is normal with deep tendon reflexes showing hyporeflexia. She has an intention tremor when she tries to pick up an object from a table and exhibits difficulty in performing the finger to nose test. What is the location of the MOST probable area of the lesion?
A. Right side, cervical spinal cord
B. Left side, cerebral hemisphere
C. Left side, substantia nigra
D. Right side, cerebellar hemisphere

A

D This patient’s signs and symptoms are reflective of a right-sided cerebellar hemisphere lesion. That is, cerebellar lesions commonly produce hypotonia, hyporeflexia, ataxia and an intention tremor. Cerebellar lesions also produced ipsilateral signs and symptoms in the extremities. A right-sided lesion of the spinal cord (Brown-Sequard syndrome) would produce weakness with spasticity and hyperreflexia. A left sided cerebral hemisphere lesion would present similar to a right-sided cervical spinal cord lesion. A lesion of the substantia nigra would produce signs and symptoms like as those seen with Parkinson’s disease with rigidity and a resting tremor

How well did you know this?
1
Not at all
2
3
4
5
Perfectly
35
Q

A patient received a gunshot wound to the distal posterior thigh that resulted in complete severance of the common fibular (peroneal) nerve. Which of the following exercises is necessary in the early rehabilitation period to prevent contracture formation?
A. Active plantarflexion, inversion, and toe extension
B. Passive dorsiflexion, eversion, and toe extension
C. Active dorsiflexion, eversion, and toe flexion
D. Passive plantarflexion, eversion, and toe flexion

A

B
The common peroneal nerve innervates the following muscles: peroneus longs and brevis, tibialis anterior, extensor digitorum longs and brevis, peroneus tertius and extensor hallucis longus. Because of the complete nerve severance, all of those muscles would be affected and would not be able to actively contract until regeneration has taken place. Therefore, the patient would completely lose ankle dorsiflexion, eversion, and toe extension. Those motions would have to be performed passively in order to maintain range of motion. Option B is the only one that contains all of the correct actions

How well did you know this?
1
Not at all
2
3
4
5
Perfectly
36
Q

A patient with a mild closed head injury and bilateral femur fractures requires instruction in a lower extremity exercise program. To plan the most effective teaching methods for this patient, what is MOST critical to assess at the initial visit?
A. Comprehension of written, verbal, and demonstrated instructions
B. Short-term memory capacity
C. Auditory and visual status
D. Any personality changes compared to the patient’s premorbid status

A

A
Option A is the best answer. In Option B, if the patient does not initially understand the information, then having short-term memory is irrelevant. For Option C, hearing and vision are obviously important, but the most critical component is comprehension. Option D is irrelevant to the scenario.

How well did you know this?
1
Not at all
2
3
4
5
Perfectly
37
Q

For a patient with a bilateral transfemoral amputation to maximize balance in a wheelchair, the rear wheels should be positioned more:
A. laterally.
B. posteriorly.
C. anteriorly.
D. inferiorly.

A

B The center of gravity of a person with bilateral above-knee amputations is more posterior than the center of gravity of a person with their lower extremities intact. Setting the back wheels more posteriorly will make the patient more stable in the chair. This adjustment prevents the wheelchair from tipping backward.

How well did you know this?
1
Not at all
2
3
4
5
Perfectly
38
Q

On day 4 of physical therapy treatment, a patient who has had an open reduction and internal fixation for a hip fracture develops a large ecchymosis over the unaffected hip. The patient has MOST likely developed
A. a hemangioma.
B. a complication of anticoagulant therapy.
C. a hematocele.
D. deep vein thrombophlebitis

A

B Anticoagulation therapy inhibits the synthesis and function of clotting factors and can lead to bruising (ecchymosis) or more severely hemorrhage. A hemangioma is a benign liver tumor.
Deep vein thrombophlebitis would result in redness, swelling and warmth in the effected area and most commonly occurs in the calf muscle, not the hip region. A hematocele is a blood cyst.

How well did you know this?
1
Not at all
2
3
4
5
Perfectly
39
Q

A patient who is exercising on a bicycle ergometer in the cardiac rehabilitation unit is being monitored with electrocardiogram leads. During the exercise, the ST segment becomes depressed by 2 mm. If the cardiologist has given no specific guidelines, the physical therapist should
A. terminate the session immediately.
B. reduce the resistance load.
C. instruct the patient to slow down
D. direct the patient to continue as before.

A

A An ST segment depression of greater than 1 mm is indicative of myocardial ischemia.
Continuing the exercise session could place the patient at serious risk for continued ischemia, which could ultimately lead to myocardial infarction. Therefore, the best response for the therapist would be to stop the exercise session immediately and inform the cardiologist of the patient’s response. All of the other options would continue to place the patient at risk

How well did you know this?
1
Not at all
2
3
4
5
Perfectly
40
Q

Prior to starting an exercise training program, a patient with cardiac problems who is receiving beta-blocking medication should receive an explanation of the:
A. greater benefits from cardiovascular exercise to be achieved at lower rather than at higher metabolic levels.
B. need to use measures other than heart rate to determine intensity of exercise
C. need for exercise training sessions to be more frequent but of shorter duration
D. need for longer warm-up periods and cool-down periods during exercise sessions.

A

B The patient taking beta-blocking medication will experience a lower heart rate and blood pressure response during exercise as compared to a patient who is not taking this medication.
Since the heart rate is lower than anticipated, using heart rate to monitor exercise intensity may not be as accurate. Another measure, such as the Borg Scale (rating of perceived exertion) would be more beneficial. However, even though heart rate and blood pressure would be lower, the patient actually achieves the same metabolic levels during exercise and therefore altering the frequency or duration of exercise is unnecessary. The time for warm-up and cool-down exercises does not need to be altered.

How well did you know this?
1
Not at all
2
3
4
5
Perfectly
41
Q

A physical therapist places electrodes on a patient to monitor surface electromyographic activity.
The electrode placement shown in the photograph is MOST appropriate to monitor which of the following muscles?
A. Tensor fasciae late
B. Sartorius
C. Rectus femoris
D. Gluteus minimus

A

A The electrode placement in the photograph is the best placement for the tensor fascia late.
The electrodes are placed over the muscle belly and lined up parallel to the muscle fibers. The sartorius is found more distal and medial and following the direction of the muscle fibers. The rectus femoris is found more distal and central to the thigh. The gluteus medius is a deep muscle that is not monitored directly by surface electromyography easily.

How well did you know this?
1
Not at all
2
3
4
5
Perfectly
42
Q

A patient with chronic adhesive capsulitis of the shoulder is to be treated with joint mobilization to increase joint range of motion. Which of the following is the MOST appropriate to increase shoulder abduction?
A. Distraction with inferior glide
B. Anterior glide with internal rotation
C. Superior glide with approximation
D. Distraction with posterior glide

A

A Since most of the joint contracture/adhesion forms in the anterior-inferior portion of the capsule, treatment to mobilize that portion of the capsule is of primary concern. Inferior glide would stretch the inferior portion of the capsule.

How well did you know this?
1
Not at all
2
3
4
5
Perfectly
43
Q

An 8 year-old child who has juvenile rheumatoid arthritis is developing bilateral knee flexion contractures. Which of the following is LEAST appropriate?
A. Exercises to increase joint mobility
B. Ultrasound to the hamstring insertions to increase tissue extensibility
C. Progressive weight-bearing by walking in a therapeutic pool to increase endurance
D. Exercise to increase quadriceps strength

A

B
Options A, C, and D are acceptable interventions for this patient. The epiphyseal areas (growth areas of bones) in children should not be exposed to ultrasound. The application of ultrasound over the knee joint (hamstring insertion) could expose the epiphyseal areas to the ultrasound

How well did you know this?
1
Not at all
2
3
4
5
Perfectly
44
Q

A physical therapist is screening a young adult patient for a possible right thoracic, left lumbar structural scoliosis. The postural deviation commonly seen with this diagnosis is:
A. spinous processes of thoracic spine rotated to the right
B. high left shoulder.
C. posteriorly protruding scapula on the left.
D. prominent right posterior rib cage.

A

D Scoliosis deformities are named relative to the convexities of the curves, with the apex of the curve defining the vertebral level. In a right thoracic, left lumbar structural scoliosis, there is a convex curve to the right in the thoracic spine and a convex curve to the left in the lumbar spine.
In the thoracic spine, the body of the vertebrae rotates to the right (convex side) and the spinous processes rotate toward the left (concave side). Along with this rotation, the ribs rotate posteriorly on the convex side causing the scapula to become more prominent on the convex side and causing the shoulder to be elevated on the convex side.

How well did you know this?
1
Not at all
2
3
4
5
Perfectly
45
Q

An exercise program for a patient with Parkinson’s disease would MOST likely include exercises to:
A. decrease movement of the upper extremities and trunk.
B. increase trunk flexor strength.
C. improve balance reactions and reciprocation
D. stretch the back extensor muscles

A

C Patients with Parkinson’s disease are characterized by rigidity, bradykinesia, tremor, lack of associated movements, impaired balance reactions and a flexed posture (kyphosis). Therefore, intervention should be directed toward full range of motion and correction of kyphotic posture through back extensor strengthening exercises, balance exercises, and exercises that promote reciprocal movement. Option C is the only one that includes appropriate exercises.

How well did you know this?
1
Not at all
2
3
4
5
Perfectly
46
Q

If a transtibial prosthesis has an excessively firm heel wedge, the patient is MOST likely to walk with:
A. excessive knee flexion in foot flat (loading response).
B. premature knee flexion in heel off (terminal stance).
C. insufficient knee flexion in foot flat (loading response).
D. excessive knee flexion in heel off (terminal stance).

A

A
Upon heel contact, the heel section compresses, partially absorbing the ground impact and thus permitting a controlled plantarflexion. If the heel is too stiff, upon initial contact, the knee flexes too soon and excessively on initial contact. An excessively firm heel wedge would not be a factor in late stance.

How well did you know this?
1
Not at all
2
3
4
5
Perfectly
47
Q

A physical therapist is planning a patient education program for a group of patients with chronic low back pain. To increase the likelihood that the patients will utilize the proper body mechanics at the work site, the therapist should:
A. ask the patients to demonstrate use of proper body mechanics.
B. provide a reference list of articles describing body mechanics
C. ask the patients to describe actions they do that increase their back pain.
D. provide information on the frequency of low back injuries due to improper body mechanics

A

A Evaluation of the patient’s retention of the information presented in the program can be enhanced by asking the patients questions about the program information, having the patients ask questions about the program, having the patients demonstrate what they have learned and testing the patient about the program material. Option A is the only one that utilizes one of these techniques. The other options do not require active participation by the patients, and they do not address specific interventions

How well did you know this?
1
Not at all
2
3
4
5
Perfectly
48
Q

A patient presents with hemiparesis and demonstrates a foot drop during the swing phase of gait.
It is MOST appropriate to administer functional electrical stimulation to the tibialis anterior and the:
A. gastrocnemius.
B. tibialis posterior.
C. extensor digitorum longus.
D. peroneus longus.

A

C
During the swing phase of gait, the foot is dorsiflexed to clear the floor. Muscles active during this phase of gait are the tibialis anterior, extensor hallucis longus, and the extensor digitorum longus. The gastrocnemius, tibialis posterior and peroneus longus are active during stance phase of gait. Therefore, of the muscles listed, the extensor digitorum longs is the best choice. If only the tibialis anterior was stimulated the foot would go into inversion and dorsiflexion, which could cause the patient to land on the lateral side of the foot at initial contact causing them injury. Addition of the extensor digitorum longs would bring the foot up into more dorsiflexion and avoid excessive inversion.

How well did you know this?
1
Not at all
2
3
4
5
Perfectly
49
Q

A physical therapist is treating a patient with iontophoresis. If the current is set at 4 mA, the duration at 15 minutes, and the dexamethasone (Decadron) solution at 0.04%, with an electrode area of 4 cm’, how should the therapist record the current dosage in the patient’s chart?
A. 0.16 mA-mg
B. 16 mA/cm?
C. 0.60 mg/min
D. 60 mA-min

A

D
The current dosage for ontophoresis is calculated by multiplying the current (4 mA) by the duration (15 minutes). In this case the calculation would yield a value of 60 mA-min. None of the other options use the correct values.

How well did you know this?
1
Not at all
2
3
4
5
Perfectly
50
Q

An elderly patient has a diabetic neuropathy. Upon examination, the patient shows marked mediolateral instability of the left ankle while walking. The patient also has fluctuating edema and glove-and-stocking sensory loss in both of the lower extremities. The MOST appropriate orthotic aid is:
A. a double-upright metal ankle-foot orthosis.
B. high-top shoes.
C. a prefabricated plastic solid ankle-foot orthosis
D. a spiral ankle-foot orthosis.

A

A
The double upright metal ankle foot orthosis (AFO) would provide the best support for the mediolateral instability and would allow for changes in leg volume. The plastic, solid AFO would not provide as much mediolateral support and because of its conformation to the leg it would not be advisable for patients who are experiencing changes in leg volume due to the edema. High top shoes would not provide enough support. The spiral AFO would not provide needed stability to the ankle and the design would not be indicated when a patient has fluctuating edema in the leg.

How well did you know this?
1
Not at all
2
3
4
5
Perfectly
51
Q

A 25 year-old patient with cystic fibrosis has been experiencing successful mobilization of thickened secretions by standard percussion and postural drainage. During the current treatment session, the patient coughs up sputum that has a small, brownish streak of blood. The physical therapist should:
A. percuss more vigorously in an attempt to clear out any other blood that is present in the airwavs
B. perform postural drainage in upright positions for this type of patient only.
C. continue treatment with little modification, unless more blood is noted.
D. stop the treatment and send the patient to the emergency room.

A

C Hemoptysis or the coughing up of blood can be benign or severe and can be a common occurrence in the cystic fibrosis population secondary to the pathophysiological processes in their pulmonary system. In general, a small amount of hemoptysis that occurs occasionally or is non-persistent hemoptysis is benign. The “brownish” color indicates either venous blood or old blood. In this situation, if the hemoptysis does not persist, then it is appropriate to continue treatment without modification. If the blood was larger in amount and fresher (denoted by a bright red color), then the treatment should be discontinued. The presence of blood in the sputum should not warrant more vigorous chest percussion, until it is deemed appropriate to continue with the session. Since this situation is a benign scenario, alteration of the drainage position is not necessary, and there is no need to send the patient to the emergency room.

How well did you know this?
1
Not at all
2
3
4
5
Perfectly
52
Q

Which of the following should be recommended for a patient following a medial meniscectomy, if the physical therapy plan is to exercise the quadriceps femoris against accommodating resistance?
A. Knee bends through half of the range
B. Knee extension with a sandbag at the ankle
C. Knee extension on an isokinetic exercise device
D. Straight-leg raising with a sling suspension

A

C
Isokinetic exercise devices utilize accommodating resistance throughout the range of motion. The other exercises are not forms of accommodating resistance.

How well did you know this?
1
Not at all
2
3
4
5
Perfectly
53
Q

A physical therapist is examining a patient who has difficulty making a fist. Results of the patient’s muscle tests show the following: Flexors of the index, middle, ring, and little fingers (2nd to 5th digits) at the proximal interphalangeal joint are Normal (5/5). Flexors of the index finger (2nd) and middle finger (3rd digit) at the distal interphalangeal joint are Normal (5/5). Flexors of the ring finger (4th digit) and little finger (5th digit) at the distal interphalangeal joint are Poor (2/5). These findings are consistent with a lesion of the:
A. median nerve C8-T1.
B. median nerve C5-C6.
C. ulnar nerve C5-C6.
D. ulnar nerve C8-T1.

A

D The weakness is in the flexors of the fourth and fifth digits at the distal interphalangeal joint (2/5). The other fingers show normal strength (5/5). This weakness would indicate a problem with the flexor digitorum profundus, since the distal joints are involved. The flexor digitorum profundus is innervated by both the median and ulnar nerves. The flexor digitorum profundus muscles flexing the index and middle fingers are innervated by the median nerve. The flexor digitorum profundus muscles flexing the ring and little fingers are innervated by the ulnar nerve.
Therefore, since the ring and little fingers are involved, the lesion would have to be involving the ulnar nerve. The flexor digitorum profundus to the fourth and fifth fingers is innervated by the ulnar nerve roots C8 and T1.

How well did you know this?
1
Not at all
2
3
4
5
Perfectly
54
Q

An elderly man who lives alone fell in his home, sustaining a hip fracture, which necessitated a total hip arthroplasty. The patient was previously able to perform all activities of daily living independently. He has been admitted to a skilled nursing facility for rehabilitation. The patient’s goal is to return home. His family believes he should be admitted to a long-term care facility for his own safety once acute rehabilitation is completed. The physical therapist’s MOST appropriate action is to:
A. advise the family that the patient should determine his own discharge environment.
B. recommend a team conference with the patient and his family to discuss discharge plans.
C. schedule a home visit to determine if home modifications are needed before discharge.
D. implement a treatment plan with a long-term goal of discharge to home.

A

B The best answer would be for the therapist to recommend a team conference. That conference would bring together members of the health care team as well as the patient and family members, to arrive at joint decision as to the placement of this patient. The best time for such a meeting would be prior to discharge, because after discharge it is difficult to arrange such meetings. Allowing the patient to determine his own discharge environment may be fine if there are no safety concerns, however a decision whether or not the patient is competent to make that decision must be made first. Scheduling a home visit or the implementation of a treatment plan for home care would be premature until a decision on placement has been made.

How well did you know this?
1
Not at all
2
3
4
5
Perfectly
55
Q

During lower extremity exercise, a patient with incomplete C6 quadriparesis reports a new onset of tingling and numbness in both hands. A manual muscle test reveals that overall strength has decreased by 1 grade. In this situation, the MOST appropriate action for the physical therapist to take is to:
A. modify the exercise program to accommodate the change in strength.
B. apply an ice massage prior to exercise
C. continue the lower extremity therapeutic exercise and use facilitation techniques to stimulate more activity.
D. discontinue all exercise and notify the physician immediately.

A

D Sudden tingling or the hands and weakness of muscle groups are alarming signs of cord compression. The concern is that the spinal cord is sustaining further injury. The exercise may be exacerbating or causing these signs. This is considered a medical emergency and the physician should be immediately advised.

How well did you know this?
1
Not at all
2
3
4
5
Perfectly
56
Q

A physical therapist is treating a patient with moderately well-controlled type 1 diabetes. One of the goals is to help the patient regulate insulin dosage through exercise. In establishing an exercise program, the therapist should be aware that regular exercise will generally:
A. lower blood glucose levels and decrease the amount of insulin required.
B. lower blood glucose levels and increase the amount of insulin required
C. raise blood glucose levels and decrease the amount of insulin required
D. raise blood glucose levels and increase the amount of insulin required.

A

A In patients with moderate hyperglycemia, exercise can lead to hypoglycemia for periods of 24 to 48 hours after exercise. Exercise has been shown to increase sensitivity of the insulin receptors therefore leading to a decrease in the amount of insulin required.

How well did you know this?
1
Not at all
2
3
4
5
Perfectly
57
Q

A physical therapist recognizes that departmental patient education materials are written on a reading level that is too difficult for some of the patients. The physical therapist can help the patients to better understand the materials by:
A. including illustrations and a larger print.
B. including illustrations and a simpler sentence structure.
C. reducing the number of syllables and lengthening the sentences.
D. using a large print format to present specific medical terminology

A

B
A larger print will not simplify the meaning, nor will lengthening the sentences. Only Option
B, the choice of illustrations and a simpler sentence structure, is correct.

How well did you know this?
1
Not at all
2
3
4
5
Perfectly
58
Q

During inspiration, a patient demonstrates increased upper chest expansion with retraction of the epigastric area. The physical therapist should suspect weakness of the:
A. scalene muscles.
B. diaphragm.
C. rectus abdominis.
D. intercostal muscles.

A

B
Normally, as the diaphragm descends with inspiration, it places increased pressure on the abdominal contents with a resultant distension of the epigastric area. Weakness of the diaphragm would decrease the distension and may result in a compensatory increase in upper chest expansion to improve ventilation. The scalene muscles act to elevate or stabilize the clavicle during inspiration. The rectus abdominus is a stabilizer during inspiration and assists with forced expiration. The intercostal muscles act to stabilize the thoracic cage during ventilation.

How well did you know this?
1
Not at all
2
3
4
5
Perfectly
59
Q

A patient with low back pain has been undergoing treatment for 2 sessions. The patient tells the physical therapist that today the pain is centralizing with the extension exercises, but is as intense as it was at the first treatment session. The patient is frustrated by this reaction. The MOST appropriate response of the therapist is to :
A. continue with the present program.
B. eliminate the extension exercises
C. consult the patient’s physician about the situation
D. progress to trunk flexion exercises

A

A
With an extension exercise program, centralization indicates that the patient is improving, even though the pain may be just as intense as it was. Even though the patient may feel frustrated, he is actually improving; and therefore the therapist should continue with the extension exercise program. There is no need to call the physician at this point, since the centralization is perfectly normal. Although trunk flexion may be used later in the intervention, it would not be indicated at this point because extension is working.

How well did you know this?
1
Not at all
2
3
4
5
Perfectly
60
Q

An attorney calls the physical therapy department seeking information on the status of his client, a patient who is undergoing rehabilitation following total knee surgery. The patient’s physical therapist should:
A. refer the attorney to the patient’s insurance carrier.
B. describe the patient’s current status.
C. require written authorization from the patient to release information.
D. obtain verbal permission from the patient to release information

A

C Information relating to the physical therapist/patient relationship is confidential and may not be communicated to a third party not involved in that patient’s care without the prior written consent of the patient. Option C is the only one that fits these criteria.

How well did you know this?
1
Not at all
2
3
4
5
Perfectly
61
Q

Involving interdisciplinary team members in making decisions about continuity of patient care has all of the following benefits EXCEPT:
A. promoting each individual’s commitment to, and responsibility for, the decisions that have been made by the group
B. discouraging 1 or 2 team members from being in control and having all the authority.
C. enabling decisions regarding quality of care to be made quickly.
D. promoting quality of care by including several aspects of the patient’s care.

A

C Inclusion of all members of the health care team into decision-making is important for the patient and provides for continuity of care for the patient. However, the team approach has some pitfalls such as arranging meetings between team members, which may be a time consuming process. The team concept would also assume that the team reaches a consensus that requires more time than relying on individual decisions that can be made quickly. Therefore, Option C is the best answer to this question. The other three options are advantages of the team concept.

How well did you know this?
1
Not at all
2
3
4
5
Perfectly
62
Q

A physical therapist examines a patient with multiple sclerosis who is in a period of exacerbation.
The patient is independent with bed mobility, can sit unassisted at the edge of the bed, and requires physical assistance to stand with a walker. Which of the following activities of daily living is the MOST important for the initial intervention session?
A. Wheelchair propulsion up a 10-ft (3-m) ramp
B. Wheelchair transfers
C. Walking with an assistive device
D. Tub transfers

A

B
Physical therapy intervention should focus on helping the patient obtain maximal functional independence. Wheelchair transfers are the means to enable the patient to be independently mobile and will be a requirement before more difficult tasks are performed such as ascending a
10-foot ramp. The patient is presumably too weak to walk at this point in time. Tub transfers are important, but the wheelchair will be the best means to get the patient to the tub. Therefore, for the initial intervention session wheelchair transfers would be the most important

How well did you know this?
1
Not at all
2
3
4
5
Perfectly
63
Q

While examining a patient who had a baby 3 days ago by vaginal delivery, the physical therapist notices that the patient has a 2-cm diastasis of the rectus abdominis. To address this problem, the patient should:
A. perform partial sit-ups supporting her abdominal muscles with her hands, while lifting her head in the supine position.
B. not perform exercises, until the diastasis heals spontaneously.
C. perform sit-ups with the knees bent and arms behind the head.
D. perform partial sit-ups with legs straight and arms in front.

A

A Raising just the head activates only the recti muscles, and supporting the abdominal muscles with the hands provides external support to the stretched abdominal muscles. This is the optimal position to initiate abdominal strengthening following delivery for a patient with diastasis rectus abdominus. Doing no exercise would not be appropriate. With no support of the abdominal muscles, Options C and D would be too aggressive at this time.

How well did you know this?
1
Not at all
2
3
4
5
Perfectly
64
Q

When evaluating wheelchair positioning of a child with cerebral palsy, the physical therapist should FIRST examine the position of the child’s:
A. pelvis.
B. lower extremities.
C. head.
D. spine.

A

A The assessment of posture in a wheelchair begins with the pelvis and its relationship to its adjacent segments. The orientation and range of mobility of the pelvis in all three planes will in turn determine the alignment and support needed at the trunk, head, and extremities

How well did you know this?
1
Not at all
2
3
4
5
Perfectly
65
Q

For a patient with a right-middle-lobe pneumonia, the proper bronchial drainage position is supine with the body:
A. one-quarter turned to the left and the foot of the bed raised 14 in (35.6 cm).
B. one-quarter turned to the right and the bed flat.
C. three-quarters turned to the left and the head of the bed raised 14 in (35.6 cm).
D. three-quarters turned to the right and the foot of the bed raised 14 in (35.6 cm).

A

A
In order to place the right middle lobe in an optimal position to drain secretions, a patient would need to be positioned as stated in Option A. Another way this position may be stated is
“left sidelying with a quarter turn towards supine”. The other positions do not optimally drain this lobe.

How well did you know this?
1
Not at all
2
3
4
5
Perfectly
66
Q

To avoid the appearance of increased motion, what movement MUST be prevented during goniometric measurement of shoulder abduction?
A. Upward rotation of the scapula
B. Medial (internal) rotation of the shoulder
C. Lateral (external) rotation of the shoulder
D. Lateral flexion of the trunk to the opposite side

A

D
During measurement of shoulder abduction, the spine should be kept straight. If the patient is allowed to laterally flex the trunk it will give the appearance of increased shoulder abduction, but the motion would be occurring in the spine and not in the shoulder. Upward rotation of the scapula and lateral rotation of the shoulder would normally accompany the motion of shoulder abduction. Medial rotation of the shoulder would decrease the available abduction

How well did you know this?
1
Not at all
2
3
4
5
Perfectly
67
Q

A patient who received an organ transplantation 4 years ago demonstrates progressive weakness and is referred to physical therapy for strengthening exercises. In reviewing the medical history, the physical therapist learns that the patient takes prednisone Deltasone) and immunosuppressive drugs. Which of the following does the therapist need to consider?
A. Isokinetic strengthening will be preferable for this patient
B. The patient may have limited range of motion from increased bone mass.
C. The patient will need to be treated in an isolation room.
D. Muscle strengthening may be limited in this patient.

A

D An adverse side effect of prolonged prednisone use, in this case for 4 years, is muscle degeneration and therefore achieving increases in muscle strength may be limited. Since muscle strengthening may be limited, then isokinetic exercise would not necessarily be more beneficial.
Prednisone may lead to a decrease in bone density, not an increase. Four years after transplantation, the patient will have been out in the community and the need for an isolation room is unnecessary.

How well did you know this?
1
Not at all
2
3
4
5
Perfectly
68
Q

The results of a developmental screening of a 4 month-old infant indicate a possible right hip dislocation. The presence of which of the following is the MOST consistent with this diagnosis?
A. Pelvic obliquity, apparent pain on movement of the right hip, and decreased hip abduction on the left side
B. Asymmetrical gluteal folds, femoral telescoping, and limited hip abduction on the right side
C. Decreased active movement of the right hip, increased femoral neck retroversion, and atrophy of the gluteus maximus on the right side
D. Hip flexion contracture on the right side, apparent leg-length discrepancy, and inability to tolerate supported sitting

A

B Physical findings of congenital hip dislocation include asymmetric skin folds of the buttocks and adductor region; limitation of passive hip abduction; leg length inequality; and, telescoping of the flexed and adducted thigh on the pelvis.

How well did you know this?
1
Not at all
2
3
4
5
Perfectly
69
Q

A 14 month-old child with brachial nerve palsy has motor and sensory loss in the right upper extremity in the areas innervated by Cs and C.. Which of the following activities is MOST difficult for the child to accomplish with the affected upper extremity?
A. Pushing a wagon
B. Carrying a teddy bear in the crook of the arm
C. Holding a raisin in the palm of the hand
D. Grasping a cup

A

B The C5 and C6 myotomes include the biceps brachi, brachialis, brachioradialis, and deltoid muscles. The elbow flexors and supinators (C5-6) are used to carry a teddy bear in the crook of the arm. Pushing a wagon requires the use of the triceps (C7-8). Holding a raisin requires presence of wrist flexors (C6-8). Grasping a cup requires finger, thumb and wrist muscles (C6-8, T1).

How well did you know this?
1
Not at all
2
3
4
5
Perfectly
70
Q

A physical therapist is treating a patient who has been diagnosed with a nerve root impingement on his right side due to a narrowing of the intervertebral foramen between La and L§. The therapist decides to use mechanical, lumbar traction as an intervention. Which of the following positions, on a traction table, is MOST appropriate to relieve pressure on the nerve root?
A. Supine with the hips and knees flexed
B. Supine with the hips and knees straight and laterally shifted to the right
C. Prone with the hips and knees straight
D. Sidelying on the right with a bolster between the table and patient

A

A In order to open up the intervertebral foramen, it would be best to flex or at least flatten the lumbar spine prior to applying the traction. Of the above options, positioning the patient supine with the hips and knees flexed would be the best way to do this. Laterally bending the trunk to the left would also open up the foramen, but this is not one of the options. Supine with hips and knees straight and then laterally shifted to the right would tend to close down the intervertebral foramen. Prone would also close the foramen. Sidelying with a bolster under the right lumber spine would also tend to close the foramen.

How well did you know this?
1
Not at all
2
3
4
5
Perfectly
71
Q

A patient who is transported to the physical therapy department in a wheelchair reports severe, bilateral lower extremity pain. A purple discoloration of both feet is observed. The pain is relieved when the patient’s feet are raised just above the horizontal plane. These signs are MOST indicative of:
A. arterial insufficiency.
B. intermittent claudication.
C. venous insufficiency.
D. a psychosomatic episode.

A

C With arterial insufficiency, elevation increases ischemia and therefore pain. Intermittent claudication is a phenomenon associated with metabolic demands exceeding the capability of the vascular system to supply adequate blood flow. Placing the limb in a dependent position increases swelling and therefore possibly pain with venous insufficiency. An objective sign such as purple discoloration rules out a psychosomatic episode.

How well did you know this?
1
Not at all
2
3
4
5
Perfectly
72
Q

A patient who completes a Phase | (in-hospital) cardiac rehabilitation program should be expected to achieve:
A. independence in self-monitored walking.
B. increased aerobic capacity to physical activity.
C. modification of risk factors.
D. increased blood pressure at a given workload.

A

A
The primary physical therapy goal of Phase I cardiac rehabilitation is to achieve a safe and independent level of activity that can be carried out at home. Increasing aerobic capacity cannot happen until Phase Ill, when cardiac tissue has healed fully. Modification of risk factors is a life-long process and will not be achieve during a short inpatient stay. Systolic blood pressure, when exercising, during the inpatient stay should only rise approximately 20 mm Hg above resting.

How well did you know this?
1
Not at all
2
3
4
5
Perfectly
73
Q

A physical therapist has been working with a patient who has a spinal cord injury. To document that the patient has been educated about skin care, the therapist should record that the patient:
A. is unable to tolerate more than 1 hour in a wheelchair.
B. is unable to tolerate more than 1 hour in a wheelchair.
C. is able to state 3 causes of skin breakdown.
D. is able to perform 10 push-ups in the wheelchair.

A

C Option C is the only one stated clearly in educational terms. The other options are objectives based on observations

How well did you know this?
1
Not at all
2
3
4
5
Perfectly
74
Q

A patient reports pain lateral to the coracoid process. When palpating the shoulder to assess the possible cause of the pain, starting at the coracoid process and moving laterally, the physical therapist should expect to find the following sequence of structures:
A. lesser tuberosity, biceps tendon, and greater tuberosity
B. greater tuberosity, biceps tendon, and lesser tuberosity.
C. lesser tuberosity, coracobrachialis tendon, and greater tuberosity
D. greater tuberosity, coracobrachialis tendon, and lesser tuberosity.

A

A
Starting at the coracoid process and moving laterally, one would palpate the following structures: (1) lesser tuberosity, (2) biceps tendon and (3) greater tuberosity. Any other order of structure palpation would be incorrect.

How well did you know this?
1
Not at all
2
3
4
5
Perfectly
75
Q

A factor that MUST be considered in designing an exercise program for a patient with angina pectoris is that when the arms are exercised:
A. angina occurs at a lower heart rate than when exercising the lower extremities.
B. heart rate and blood pressure rise more steeply in relation to workload than when exercising the lower extremities
C. pulmonary ventilation increases less rapidly than when exercising the lower extremities.
D. angina occurs at higher workloads than when exercising the lower extremities

A

B
Heart rate and systolic blood pressure responses are higher for any given workload when performed with the upper extremities as compared to the lower extremities. Angina is determined by a certain myocardial workload that is represented by the product of heart rate and systolic blood pressure. Therefore Option A cannot be a conclusive statement. Pulmonary ventilation has less definitive responses than the cardiac responses but in general would be similar in that upper extremity exercise would cause a greater ventilatory response than lower extremity exercise would. Since heart rate and blood pressure would be higher at any given workload with upper extremity exercise, then angina would most likely occur at a lower workload

How well did you know this?
1
Not at all
2
3
4
5
Perfectly
76
Q

A physical therapist is treating a patient for limitation of motion following knee surgery several weeks ago. The patient’s passive knee extension is lacking 15° from full extension, and knee flexion is limited to 95°. Both movements have a capsular end-feel. Which of the following mobilization techniques is MOST appropriate for increasing knee flexion?
A. Anterior glide of the tibia on the femur
B. Posterior glide of the tibia on the femur
C. Superior glide of the patella
D. Posterior glide of the femur on the tibia

A

B
During normal knee extension, the tibia moves posterior relative to the femur. Therefore, posterior gliding of the tibia would promote knee flexion. Superior glide of the patella could be used to increase knee extension. Posterior glide of the femur on the tibia would be used to increase knee extension

How well did you know this?
1
Not at all
2
3
4
5
Perfectly
77
Q

When considering special tests for orthopedic assessment of a client, the validity of a test is important because the:
A. results of the test can be standardized.
B. test measures what it is supposed to measure.
C. results of the test are reproducible
D. test can be accurately performed by someone else.

A

B Option B is the definition of validity. Options C and D refer to reliability measures. Results can be standardized even though they are not valid.

How well did you know this?
1
Not at all
2
3
4
5
Perfectly
78
Q

A patient sustained a Colles’ fracture 8 weeks ago and has been in a cast since that time.
Immediately after cast removal, it is MOST appropriate for the physical therapy intervention for the wrist and hand to include:
A. passive and active assistive range of motion exercises.
B. progressive resistive exercises.
C. grade-4 joint mobilization techniques.
D. return to prefracture level of activity.

A

A
The primary physical therapy goal at this time is to restore range of motion. Therefore, the most appropriate intervention for that goal is passive and active assistive ROM exercises.
Although gentle joint mobilization techniques may be indicated, grade 4 techniques at this time would not be. Progressive resistive exercises would come later in the plan of care. Although the long-term goal would be to return to normal activities, the short-term goal would not

How well did you know this?
1
Not at all
2
3
4
5
Perfectly
79
Q

A patient with a traumatic brain injury is receiving outpatient physical therapy. The physical therapist notices that the patient becomes agitated during therapy sessions. To which of the following professionals should the patient be referred for assessment and diagnosis?
A. Occupational therapist
B. Neuropsychologist
C. Vocational counselor
D. Speech-language pathologist

A

B Because the problem is a behavioral disorder, the most appropriate person to assess this patient would be a neuropsychologist. The neuropsychologist is trained to assess intellectual, emotional and coping functions of the patient as well as levels of depression and anxiety. They could also provide consultation to the rehabilitation team members as to how to deal with the problem.

How well did you know this?
1
Not at all
2
3
4
5
Perfectly
80
Q

During examination of the jaw-opening pattern of a patient with a temporomandibular joint problem, the therapist notes early protrusion of the mandible. Which of the following mandibular movements MOST likely causes the protrusion?
A. Condylar translation
B. Mandibular depression
C. Condylar rotation
D. Lateral glide

A

A The protrusion component involves the arthrokinematic movement of anterior condylar translation. Mandibular depression (jaw opening) involves both condylar rotation and anterior translation. However, this question is asking only about the protrusion component. Lateral glide involves anterior translation on the contralateral side and spin on the ipsilateral side.

How well did you know this?
1
Not at all
2
3
4
5
Perfectly
81
Q

A child who has cerebral palsy has been receiving physical therapy 1 time/week for the past month. During the fourth visit, the physical therapist instructs the parents in a daily exercise program for the child, including stretching techniques. The BEST method to ensure correct implementation and carry-over of the program is for the therapist to:
A. reassess the patient’s range of motion 1 month after instructing the parents.
B. review the parents’ written notes on the child’s progress.
C. demonstrate exercises for the parents and ask them to return for a program check in 1 week.
D. ask the parents to demonstrate the exercises and to return for a program check in 1 week.

A

D The therapist should observe the parents performing the program as instructed to ensure correct implementation. The parents should return in one week for another demonstration to determine if they can correctly repeat the program as previously instructed.

82
Q

Excessive upward rotation of the right scapula is noted when a patient attempts to perform shoulder flexion. Which of the following exercises is MOST appropriate to help correct the excessive scapular rotation?
A. Right scapular protraction against resistance with the right arm at 90° of flexion
B. Bilateral scapular elevation with the upper extremities at 180° of flexion
C. Wall push-ups with an isometric hold at end range with the elbows extended
D. Bilateral scapular adduction with the upper extremities medially (internally) rotated and adducted across the back

A

D Excessive upward rotation of the scapula can result from weakness of the rhomboids and latissimus dorsi (downward rotators). The scapular adduction with medial rotation and adduction of the arm would require action by those muscles. Option A would help strengthen the serratus anterior, an upward rotator of the scapula. Option B would activate the upper trapezius as well as the rhomboids and since the upper trapezius is also an upward rotator of the scapula, this would not be the best exercise to use. Option C would also help strengthen the serratus anterior, which would tend to aggravate the problem

83
Q

A physical therapist is measuring ankle range of motion in a patient. The measurement shown in photograph A is greater than that measurement shown in photograph B. The MOST likely cause of the difference in measurement is:
A. Talocrural joint capsule restriction.
B. Weakness of the ankle dorsiflexors.
C. Tightness in the hamstrings.
D. Tightness in the gastrocnemius

A

D Capsular restriction would show up in both measurements. Dorsiflexor weakness would show up in both measurements. Hamstring tightness would affect knee range of motion, not ankle range of motion. In photograph A, there is more dorsiflexion present with the knee bent. In this position the gastrocnemius is on slack across the knee joint, which allows greater range of dorsiflexion. In photograph B, the gastrocnemius is stretched over the knee joint, so the ankle joint and tightness in the gastrocnemius would restrict ankle dorsiflexion.

84
Q

A patient with left hemiparesis is being discharged from a rehabilitation center. The physical therapist has been unsuccessful in getting the family’s cooperation to instruct them in transfers, bed mobility, and safety precautions. The MOST appropriate action for the physical therapist to take is to:
A. educate the patient so she can tell others how to assist her.
B. contact social services and arrange for a team and family meeting
C. refer the patient for home care and document the appropriate interventions.
D. write a home program and give it to the family.

A

B Ofthe options above, a social worker (social services) would be the most appropriate person to intervene in this case. The social worker is trained to help resolve family issues that arise and would be the best person to lead a team of caregivers and family members in deciding the appropriate action in this case. Educating the patient would only put more burden on them without guaranteeing results. A referral for home care may be appropriate, but the social worker would be the best person to make such a contact. Writing a home program for the family would not address the need to instruct family members in person.

85
Q

A patient is being examined for medial epicondylitis. With this diagnosis, the physical therapist should expect to MOST likely find pain over the:
A. origin of the flexor digitorum profundus with resisted finger flexion.
B. origin of the pronator teres muscle with active pronation.
C. medial epicondyle with passive wrist flexion.
D. insertion of the triceps brachii with passive elbow extension.

A

B The lesion is most likely a tendinitis involving a muscle or muscles that originate from the medial epicondyle of the humerus (i.e., pronator teres, palmaris longus, flexor carpi radialis and ulnaris, and flexor digitorum superficialis). Pain would be elicited with active contraction of the involved muscle (or muscles) or when the muscles) is/are passively stretched. Resisted wrist flexion and pronation would cause pain over the origin of the pronator teres. Options A, C and D do not meet the criteria for eliciting pain.

86
Q

In treating a patient who has had recurrent anterior shoulder dislocation, the physical therapist should AVOID which of the following extreme shoulder motions?
A. Adduction and lateral (external) rotation
B. Abduction and lateral (external) rotation
C. Hyperextension and medial (internal) rotation
D. Abduction and medial (internal) rotation

A

B
The mechanism that creates an anterior dislocation of the shoulder is forced abduction and lateral (external) rotation of the shoulder. This frequently results in a tear of the anterior portion of the capsule. Abduction and external rotation may subject the patient to recurrent subluxations and/or dislocations, and should therefore be avoided.

87
Q

During the examination of a patient with carpal tunnel syndrome, the physical therapist is MOST likely to find:
A. paresthesia of the medial palmar surface ofthe hand
B. weakness of finger extension of the lateral 3 digits.
C. paresthesia of the lateral 3 digits.
D. weakness in wrist flexion and ulnar deviation

A

C In carpal tunnel syndrome there is pain and paresthesias in the median nerve distribution of the hand, which includes the lateral three digits. There is weakness of the abductor pollicis brevis, but not of the wrist flexors or finger extensors.

88
Q

A patient with Guillain-Barre syndrome experiencing progressive paralysis is admitted to the intensive care unit. As a member of the interdisciplinary team managing this patient’s care, the MOST appropriate physical therapy intervention is for the physical therapist to:
A. alter the patient’s mechanical ventilation settings.
B. develop the patient’s medication schedule.
C. perform airway clearance techniques.
D. discuss the patient’s medical prognosis with the family.

A

C The key phrase in this question is “interdisciplinary.” Of all the options provided, only airway clearance is within the scope of practice for physical therapists. Patients with Guillain-Barré syndrome mav experience respiratory muscle fatigue or paralysis and are susceptible to pulmonary infection. Pulmonary hygiene is a critical role in their care.

89
Q

A patient with complete Ca quadriplegia is working on a program to increase tolerance to the upright position. While on the tilt table, the patient begins to have a pounding headache with flushing and profuse sweating. The physical therapist should FIRST:
A. lower the tilt table to a flat position.
B. remove the patient from the tilt table and return to room.
C. check the patient’s catheter.
D. check the patient’s blood pressure.

A

C
These are signs of autonomic dysreflexia. Among the most common cause is a distended or irritated bladder. The FIRST step is to remove the noxious stimulus.

90
Q

A 6 year-old child with spastic diplegia is walking in the parallel bars. The child walks with increased trunk and hip flexion. What is the MOST appropriate assistive device for this patient?
A. Standard walker
B. Forearm crutches
C. Posterior rolling walker
D. Bilateral quad canes

A

C The use of a posterior walker has been found to encourage a more upright posture during gait and to promote better gait characteristics than does the use of an anterior walker. A standard walker, forearm crutches and bilateral quad canes all emphasize trunk and hip flexion, which is already increased for this child.

91
Q

The physical therapist is teaching a patient with T12 paraplegia to fall. The patient walks with bilateral knee-ankle-foot orthoses and bilateral forearm cuff crutches using a swing-to gait. The MOST appropriate method to use to prevent injury is to:
A. have the patient wear a helmet and practice on a soft padded surface.
B. give the patient written instructions and illustrations on falling.
C. demonstrate and guide the patient through the correct process of falling.
D. demonstrate the correct method of falling.

A

C If patients are skilled in falling, they are less likely to become injured during a fall. The patient demonstrating or performing the task best verifies understanding of a skill. To accomplish this, the patient should be given a demonstration of proper falling technique and then the patient should be allowed to practice while being guided and guarded by the therapist.
Without first demonstrating and then allowing the patient to practice, the patient could be placed at risk for injury. Therefore, Options A, B, and D are not adequate to ensure patient safety.

92
Q

A physical therapist is designing an exercise program for a healthy elderly individual. Which of the following types of exercises is MOST stressful to the cardiovascular system of this patient?
A. Upper-limb ergometry
B. Spinal range-of-motion exercise
C. Quadriceps setting exercise
D. Lower-limb diagonal proprioceptive neuromuscular facilitation patterns

A

A
Heart rate and systolic blood pressure responses are higher for any given workload when performed with the upper extremities as compared to the lower extremities. Spinal motion does not create much stress on the cardiovascular system. The quadriceps setting and diagonal patterns are performed with the lower extremities and would produce less cardiovascular stress.

93
Q

A physical therapist is evaluating a patient for a wheelchair prescription. The patient, who lives alone in a small apartment, has bilateral lower extremity weakness and is unable to stand to transfer. Trunk strength and upper extremity strength are normal and the patient can transfer independently with a sliding board. The MOST appropriate prescription for this patient is a wheelchair with:
A. fixed, adjustable-height arm rests and detachable foot rests.
B. removable arm rests and detachable swing-away leg rests.
C. fixed arm rests and removable leg rests.
D. desk-style removable arm rests and fixed, elevating leg rests.

A

B
In order to facilitate transfers utilizing a sliding board, the patient will need both removable armrests and detachable swing-away leg rests. Fixed arm rests make sliding board transfers extremely difficult and unsafe. Any of the other options would not be optimal for this particular patient’s needs.

94
Q

A patient who is on bed rest and who exhibits orthostatic hypotension upon standing will MOST
likely experience:
A. increased venous tone in the lower extremities.
B. inadequate ventricular filling during diastole.
C. decreased hydrostatic pressure in the capillary beds.
D. parasympathetic stimulation of the heart.

A

B Orthostatic hypotension from bed rest occurs as a result of decreased venous tone, which will lead to a pooling of blood in the lower extremities upon standing. The pooling of blood in the lower extremities will reduce the amount of blood returning to the heart decreasing ventricular filling and ultimately decreasing cardiac output. This results in a drop in blood pressure with resultant dizziness. With pooling of blood in the lower extremities, the hydrostatic pressure would increase. To compensate for the decrease in cardiac output, the sympathetic system (not the parasympathetic) would stimulate the heart.

95
Q

Which of the following is the MOST appropriate intervention for a patient with juvenile rheumatoid arthritis who is experiencing painful swelling of both knees?
A. Resistive exercises
B. Stretching to prevent contractures
C. Gentle, active exercises
D. Walking program

A

C All of the options, except for gentle, active exercises, are precautions or contraindications for this patient.

96
Q

A patient who has left hemiparesis exhibits shoulder subluxation on the affected side. The patient has a demand-type cardiac pacemaker. Which of the following interventions should be used
ONLY with extreme caution?
A. Manual resisted exercise on the nonaffected side
B. Active exercise on the affected upper extremity
C. Functional electrical stimulation to the affected side
D. Biofeedback to the affected side

A

C Electrical stimulation for patients with demand-type pacemakers is often listed as a contraindication. However, even though it is often listed as a contraindication in textbooks, research studies apparently have not substantiated the suspected risks. Nonetheless, it should only be applied with caution and close supervision in these patients. Exercise, unless very strenuous, would be indicated in this patient. Biofeedback introduces no electrical signals into the bod and therefore would not be contraindicated

97
Q

A 5 year-old patient with a closed head injury is exhibiting behavior consistent with the confused/agitated level of cognitive function. During physical therapy intervention, the patient becomes combative. Which of the following techniques would be MOST effective in calming the patient?
A. Providing brief moving touch to the face and turning on the patient’s favorite musical tape
B. Maintaining touch to the shoulders and brightening the room by opening the curtains
C. Removing the patient from the current treatment environment and placing the patient in a quiet setting
D. Wrapping the patient in a blanket and rocking the patient rapidly

A

C Sensory inputs such as intermittent music and touching the face, rocking the patient rapidly, and brightening the room elicit arousal. When a child becomes agitated and confused, it is appropriate to reduce the general amount of environmental stimulation. Decreasing auditory and visual activity in the room may help the child focus.

98
Q

The MOST appropriate therapeutic exercise to stretch the neck muscles for a patient with an acute, right-sided torticollis is:
A. right rotation and right lateral flexion.
B. left rotation and right lateral flexion.
C. left rotation and left lateral flexion.
D. right rotation and left lateral flexion.

A

D Isolated contraction of the right sternocleidomastoid muscle would cause the head to rotate to the left and side bend (lateral flexion) to the right. To stretch the muscle, the opposite actions would be performed on the patient by the therapist: rotation of the head to the right and lateral flexion to the left.

99
Q

A patient presents to physical therapy with low back and leg pain, with symptoms extending to the bottom of the foot. During the physical therapy examination, the patient does not report leg pain in the first test position (photograph #1), but reports a severe increase in symptoms in the second test position (photograph #2). These findings support which of the following conclusions?
A. The pain is the result of a herniated disc.
B. The patient may be displaying nonorganic symptoms.
C. The symptoms are the result of an inflamed sciatic nerve.
D. The hamstrings are in a facilitated state of contraction.

A

B Because sitting knee extension and the straight-leg raise culminate in essentially identical positions, symptomatic responses to the 2 types of maneuvers should be similar. If the patient had a symptomatic herniated disc, both positions would result in a similar symptom increase.
Symptomatic responses to the 2 types of maneuvers should be similar. If the patient had an irritated sciatic nerve, both positions would result in a similar symptom increase. If the patient had a facilitated hamstring, both positions would result in a similar symptom response.

100
Q

A patient who is re-learning the task of moving from sit to stand following traumatic brain injury is frustrated because of repeated failed attempts. To facilitate the patient’s success, the physical therapist should FIRST do which of the following?
A. Permit the patient to rest until the next physical therapy session and re-attempt the activity.
B. Encourage the patient to visualize success with the task, before resuming attempts.
C. Provide incentive by holding a desired object for the patient to reach toward.
D. Decrease the challenge of the task, so that the patient experiences success

A

D It is most important for the patient to experience some form of success in order to provide motivation. Stopping the session upon failure may further frustrate the patient. Visualization, although useful, is a higher level task that should not be the first strategy used. Poor body mechanics and stimulation of tone may occur if the patient reaches forward while moving from sit-to-stand. Necessary to learning are motivation to try the unknown and, simultaneously, success in learning, to retain the learner’s motivation.

101
Q

The result of which of the following nerve tension tests is MOST likely to be positive for a waiter who has hand pain when carrying trays overhead?
A. Ulnar
B. Median
C. Radial
D. Musculocutaneous

A

A
The overhead positioning of carrying food trays is similar to the end position of the ulnar nerve tension test. The test for the ulnar nerve includes shoulder depression, abduction and external rotation, elbow flexion, forearm pronation or supination, and wrist and finger extension.
The median nerve tension test employs elbow extension, but the position of the waiter is with elbow flexion. The radial nerve test is low by the side, not reaching overhead. The musculocutaneous nerve does not innervate the hand.

102
Q

A patient reports dizziness and blurred vision when walking and turning the head to either the left or the right. The patient has no problem when the head is kept still while walking. Which of the following systems is MOST likely involved?
A. Visual
B. Vestibular
C. Somatosensory
D. Musculoskeletal

A

B Since the patient does not have symptoms while looking straight ahead, the visual acuity (peripheral visual system) does not appear to be at fault. When a patient complains of visual blurring with head motion, the patient is experiencing oscillopsia. Oscillopsia is a visual instability with head movement in which images appear to move or bounce. It is often due to decreased vestibulo-ocular reflex. No information is given in the question, which suggests that the patient is having difficulty with somatosensory input or that indicates musculoskeletal deficits.

103
Q

Which of the following questions is the MOST appropriate for a functional outcome measurement tool?
A. “How high can you lift your arm?”
B. “How long have you had this pain?”
C. “How often do you walk with a limp?”
D. “How much does this leg bother you?”

A

C Option A does not ask about function. It is an impairment-related question, not a function-related question. Options B and D do not ask about function and even with improvement, neither would relate to function. Option C is taken from the Short Musculoskeletal Function Assessment. It addresses function and can be used again as an outcome question

104
Q

A patient describes bilateral posterior lower leg aching that resolves with sitting and is exacerbated by walking, especially down hills. What diagnosis is MOST likely responsible for this pain?
A. Lateral spinal stenosis
B. Central disc herniation
C. Bilateral piriformis syndrome
D. Neoplastic spinal lesion

A

A Neurogenic claudication may be unilateral or bilateral. This scenario is bilateral. The diagnosis of lateral spinal stenosis is supported because extension increases neurological signs and flexion decreases neurological signs, regardless of weight-bearing factors. Walking down hills is worse for the patient, because the extension of the lumbar spine is greater. Disc derangements tend to be worse with flexion (sitting) and better with walking, and are rarely bilateral. Piriformis syndrome, although when present can result in sciatic pain, is rarely bilateral.
Also, walking uphill would probably be more difficult than downhill for an irritated piriformis.
Nothing in the question indicates neoplasm. The scenario seems to indicate a musculoskeletal problem, since the pain changes with position.

105
Q

A patient, who is a tennis player, has been receiving physical therapy intervention following an anterior capsular reconstruction of the dominant shoulder. The physical therapist determines that the patient is ready to begin dynamic stabilization exercises. Which of the following is an appropriate dynamic stabilization exercise for this patient?
A. Hitting a tennis ball against a wall using a forehand stroke
B. Practicing slow forehand strokes with elastic tubing attached to the racquet grip
C. Maintaining a follow through position while the therapist provides rhythmic stabilization resistance
D. Performing push-ups against the wall with emphasis placed on scapular protraction

A

A Dynamic stabilization exercises involve unconscious control and loading of the joint. They introduce ballistic and impact exercises to the patient. Practicing the forehand stroke by hitting a ball against a wall incorporates these principles. Practicing slow forehand strokes with elastic tubing attached to the racquet grip will provide a functional exercise, but not a dynamic exercise as it would not introduce ballistic movements, nor load the joint in the same way, as this motion is described as a slow movement which limits its dynamic characteristics. Maintaining a follow through position while the therapist provides rhythmic stabilization resistance is a hold position that’s isometric, not dynamic. There is no unconscious or ballistic component. Performing push-ups against the wall with emphasis placed on scapular protraction may be a good exercise, but it does not fit the criteria for being a dynamic stabilization exercise. It does load the joint, but there is no unconscious or ballistic component

106
Q

Which of the following ranges of motion is MOST important to obtain for an individual with a T42 spinal cord injury to walk?
A. Neutral dorsiflexion
B. Full hip extension
C. Normal knee flexion
D. 110° straight-leg raise

A

B
Ten degrees of dorsiflexion is required for normal walking. Full hip extension is required for walking for a person with a T12 level injury. Normal knee flexion is not required for normal walking.
While option D is important for long-sitting and floor to wheelchair transfers, this is not required for gait.

107
Q

The physical therapy intervention of a patient with dark skin pigmentation includes superficial heat. After 5 minutes of this intervention, the physical therapist removes the heat and observes that the skin over the region being treated is blotchy with both darker and lighter areas. What is the MOST appropriate action for the physical therapist to take?
A. Discontinue superficial heat and initiate ultrasound.
B. Continue with the use of superficial heat, as this is a normal response.
C. Discontinue superficial heat and initiate diathermy.
D. Continue with the use of superficial heat, but add more towel layers.

A

D There is no reason to suspect that the modality must be changed. The first logical step is to increase toweling. The patient’s response is not a normal response. An uneven blotchy red and white in light skinned, or darker and lighter areas in darker skinned patients, could be a sign of overheating. Five minutes after initiating heating, the physical therapist should remove the pack and check the patient’s skin for any blotching-mottled erythema. The heat is too intense, so adding more towel layers is the most appropriate action

108
Q

A 50 year-old patient had an uncomplicated open repair of a rotator cuff tear 2 weeks ago. The patient asks the physical therapist when the shoulder will be normal again. Which of the following expected outcome time frames MOST accurately addresses this patient’s question?
A. 3 weeks to lift 5-Ib (2.3-kg) objects
B. 3 weeks to sleep on the involved side
C. 3 months to lift the upper extremity overhead to reach into a cabinet
D. 3 months to play golf

A

C Three weeks is too early to lift 5-Ib (2.3-kg) objects. Three weeks is too early to sleep on the involved side. Usually by 8 to 12 weeks, a patient who has had an uncomplicated open repair of a rotator cuff 2 weeks ago is able to actively elevate the arm to functional heights. Three months is too early to play golf.

109
Q

A patient who had arthroscopic knee surgery 6 weeks ago currently has passive knee range of motion of 25° to 125° with pain at the end of the available range. Which of the following joint mobilization procedures is MOST appropriate for this patient?
A. Large amplitude oscillations performed within the range of motion, moving the tibia anteriorly on the femur
B. Small amplitude oscillations performed at the limit of the available motion and into tissue resistance, moving the tibia posteriorly on the femur
C. Small amplitude oscillations performed at the limit of the available motion and into tissue resistance, moving the tibia anteriorly on the femur
D. Large amplitude oscillations performed within the range of motion, moving the tibia posteriorly on the femur

A

C Option A describes a grade 2 oscillation, which is insufficient to gain range. Option B is incorrect because it describes the opposite direction. Option C is incorrect, because a grade 4 mobilization (as described) is needed to increase range of motion. Knee extension is the primary concern, requiring an anterior glide of the tibia on the femur, but option B describes the opposite direction, and is therefore incorrect.

110
Q

The work modification shown in the photograph is MOST appropriate for a patient with which of the following pathologies?
A. Central lumbar stenosis
B. Deep vein thrombosis
C. Thoracolumbar scoliosis
D. Lumbar disc bulge

A

D A patient with stenosis will do better in sitting, not standing. A patient with a deep vein thrombosis will need to move, not be stationary. A scoliosis alone does not warrant a standing work station. The photograph shows a standing desk/work station. Sitting increases intradiscal pressure, so standing is often preferred to sitting.

111
Q

A patient had knee surgery 4 weeks ago. Which of the following descriptions represents the correct neuromuscular electrical stimulation parameters to use for strengthening the quadriceps muscle of this patient?
A. 1 to 4 pps, 100 microseconds
B. 1 to 4 pps, 350 microseconds
C. 40 to 50 ps, 350 microseconds
D.100 pps, 100 microseconds

A

C The pulse width of 100 makes it difficult to achieve a strong enough motor response. A 1 to 4 pps frequency may lead to a twitch motor response, but not tetany, which is needed for strengthening. The correct answer requires a frequency that will lead to tetany (something above about 30 Hz), and a long enough pulse width to recruit motor fibers (usually something greater than 200 microseconds). Therefore, of the options, 40 to 50 ps with a pulse width of 350 microseconds is the best. Option D describes the typical parameter for conventional transcutaneous electrical nerve stimulation for acute pain management. The pulse width of 100 makes it difficult to achieve a motor response and he higher frequency of 100 ps leads to rapid fatigue.

112
Q

In order to manually assess a patient’s lower extremity circulation, a physical therapist should palpate the patient’s peripheral pulse at which of the following locations?
A. Dorsal foot, near the base of the 1st metatarsal
B. Lateral lower leg, just posterior to the fibular head
C. Lateral ankle, just inferior to the lateral mallelus
D. Plantar foot, just medial to the medial calcaneal tuberosity

A

A
The therapist should palpate the dorsal pedal pulse, which is found on the dorsal aspect of the foot near the base of the first metatarsal. The anatomical locations in options B, C, and D are not appropriate to palpate the dorsal pedal pulse.

113
Q

A physical therapist is evaluating a 70 year-old female patient who reports the onset of midthoracic pain after working in a garden for several hours. The presence of which of the following history items should increase the therapist’s suspicion of a thoracic compression fracture in the patient?
A. Bowel and bladder dysfunction
B. Smoking and prolonged steroid use
C. Hypertension and diabetes
D. Emphysema and hormone replacement therapy

A

B Bowel and bladder dysfunction are related to spinal cord/upper motor neuron dysfunction.
Smoking and steroids can weaken bone and increase the likelihood for a compression fracture.
Hypertension and diabetes are not risk factors for a compression fracture. Although emphysema is a pulmonary disease, by themselves, neither emphysema, nor hormone replacement therapy, increases the likelihood of a compression fracture.

114
Q

Which of the following sleeping positions is LEAST appropriate for a physical therapist to recommend for a patient with right glenohumeral joint anterior/inferior hypermobility and subacromial impingement?
A. Supine
B. Prone
C. Right sidelying
D. Left sidelying

A

C Often the supine position is not tolerated with an anterior hypermobility, as the humeral head is pushed anteriorly somewhat in this position. Prone is neither a good nor bad position, as far as this scenario is concerned. It would be difficult for this patient to get the shoulder in a comfortable position while lying prone. Right sidelying would be painful and could compromise circulation to the patient’s joint. It could also stress the capsule with an underlying hypermobility.
Of all the positions listed, left sidelying would be most well tolerated by this patient.

115
Q

Which of the following joints is indicated by the arrow in the radiograph?
A. Tibiofibular
B. Subtalar
C. Talocrural
D. Midtarsal

A

C
The joint indicated in the radiograph is the talocrural (ankle) joint.

116
Q

Which of the following positions is BEST to assess the length of a patient’s rectus femoris muscle?
A. Sidelying with tested hip in flexion
B. Supine with tested hip and knee in flexion
C. Prone with tested knee in flexion
D. Sidelying with tested hip in extension

A

C Option A does not mention the knee position and has the hip flexed, which shortens the rectus femoris. Option B has the hip flexed, which shortens the rectus femoris. Prone with the knee in flexion keeps the hip in neutral and does not allow the hip to flex. It describes the Ely test.
Option D has the hip in the correct position, but does not mention the knee position.

117
Q

A patient with an L - Ls posterolateral herniated nucleus pulposus is MOST likely to have sensory deficits in which of the following locations?
A. Medial knee
B. Medial ankle
C. Plantar aspect of the foot
D. Dorsum of the great toe

A

D The medial knee is the La dermatome. The medial ankle is La dermatome. The plantar foot is
S1 or S2. A posterolateral disc bulge at L4- L5 will most likelv affect the L5 nerve root and the dermatome for the Ls nerve root. The L5 dermatome includes the dorsal aspect of the great toe.
The dorsal area of the great toe is consistently included in the L5 dermatome.

118
Q

Which of the following descriptions describes the correct physical examination technique to assess tissue hydration in the hand?
A. Pinch and lift the skin and determine the time for the skin to return to normal.
B. Push into the skin and determine the time for the skin to return to normal.
C. Measure the surface temperature over the volar aspect of the wrist.
D. Obtain volumetric measurements on each arm and compare displacement measures.

A

A Tissue hydration is determined by pinching and lifting the skin and timing the return to normal. Option B describes the measurement technique for pitting edema. Temperature is not directly related to tissue hydration. Measurement of girth/size is not related to tissue hydration.

119
Q

A patient’s standing posture is characterized by a right thoracolumbar scoliosis. The scoliosis is not present when the patient is sitting. Which of the following dysfunctions is MOST likely the cause of this patient’s scoliosis?
A. Lumbar facet dysfunction
B. Unilaterally weak gluteus medius
C. Short iliopsoas muscle
D. Leg-length discrepancy

A

D A facet dysfunction would remain in sitting and in standing. It is unlikely that a postural problem present in standing is due to muscular weakness. A weak gluteus medius would display more problems during gait or movement. Although the iliopsoas is passively shortened in sitting, a short iliopsoas is not a common cause to scoliosis, especially not thoracolumbar scoliosis.
Short iliopsoas muscle is correct because when the patient is sitting, leg length does not affect the spinal posture.

120
Q

To obtain informed consent for an intervention, the physical therapist must disclose to the patient all of the following EXCEPT the:
A. cost of the intervention.
B. goals of the intervention.
C. possible risks of the intervention.
D. alternative interventions available.

A

A
Option A is not among the information that must be disclosed to a patient to obtain informed consent. The goals of the intervention, the possible risks of the intervention, and any alternative interventions available must be disclosed to the patient to obtain the patient’s informed consent.

121
Q

Which of the following statements is an appropriately written short-term goal?
A. In 2 weeks, the patient will increase walking distance from 50 ft to 100 ft (15 m to 30 m) with a wheeled walker.
B. The patient will walk 100 ft (30 m) with a wheeled walker and minimal assistance in 3 weeks
C. In 4 weeks, the patient will walk with minimal assistance 100 ft (30 m) without loss of balance.
D. The patient will increase walking distance from 50 ft to 100 ft (15 m to 30 m) with a wheeled walker independently by discharge.

A

B
Option A does not specify the amount of assistance. Option B is a measurable, specific, and objective goal. Option C does not specify the device the patient will be using. Option Dis a goal, which is to be achieved by discharge, a long- term goal, not a short-term goal.

122
Q

Which of the following pulmonary function tests measures the average total amount of air moved during inspiration of normal breathing?
A. Vital capacity
B. Inspiratory capacity
C. Tidal volume
D. Inspiratory reserve volume

A

C Vital capacity is measured from maximum inhalation to maximum exhalation. Inspiratory capacity measures volume with maximal inhalation. Tidal volume measures total volume of air moved during either inhalation or exhalation over a specific period of time (usually 1 minute) and then divided by the ventilatory rate. Inspiratory reserve volume measures inspiratory volume beyond normal inspiration.

123
Q

A physical therapist is evaluating a patient who had a right lower lobe resection due to lung cancer yesterday. During auscultation of the patient’s lungs, the therapist notes decreased low-pitched crackles bilaterally. The patient’s vital signs are: heart rate - 99 bpm; blood pressure
- 115/75 mm Hg; and pulse oximetry - 92% on 2 liters of oxygen while sitting at the edge of the bed.
Which of the following actions should the therapist take NEXT?
A. Begin walking for the patient on 4 liters of oxygen
B. Contact the physician.
C. Perform active range of motion exercises with the patient at bedside
D. Initiate bronchopulmonary hygiene

A

D Although walking usually helps to clear the lungs, the pulse oximetry is too low, although it might be okay for someone with chronic obstructive pulmonary disease, to initiate until the patient’s lungs are cleared first. The therapist should clear the lungs first, not contact the physician. If the therapist is completely unsuccessful with clearing the lungs and the pulse oximetry remains low, then contacting the physician may be appropriate. Range of motion exercises are important post-thoracotomy, but again, the patient’s breathing should be addressed first. The crackles and low pulse oximetry indicate that the patient needs better ventilation and secretion clearance, probably secondary to the recent surgery.
Bronchopulmonary hygiene, including postural drainage, percussion, vibration, suctioning, and incentive spirometry, are beneficial in treating and preventing postoperative atelectasis.

124
Q

A physical therapist is treating a patient who sustained a complete spinal cord injury. The patient currently walks with bilateral knee-ankle-foot orthoses and forearm crutches. The patient asks the therapist about how to correctly descend 1 step. Which of the following actions is MOST appropriate for the therapist to take?
A. Instruct the patient to approach the step facing forward, lower the legs to the next step, and then place the crutches on that step.
B. Instruct the patient to approach the step facing forward, place the crutches on the lower step, and then lower the legs to that step.
C. Explain to the patient that descending stairs is not an appropriate goal at this time.
D. Instruct the patient to approach the step facing forward, place one crutch on the lower step, lower the legs to that step, and then place the other crutch on that step.

A

A
Regarding option A, the legs need to be lowered first, then the crutches, otherwise the hips would buckle when the patient leans forward to lower the crutches to the next step. Regarding option B, lowering the crutches first would cause the hips to buckle. When a patient is in knee-ankle-foot orthoses and has a spinal cord injury, the patient does not have sufficient hip control, otherwise the patient would be in ankle-foot orthoses. If the patient is able to walk in knee-ankle-foot orthoses, the patient should be able to learn how to descend 1 step. Regarding option D, having the crutches on separate steps would cause the patient to be off balance and may cause the hips to unlock.

125
Q

A physical therapist is planning a 3-session educational series on physical therapy evaluation of the low back for fourth year medical students. Which of following is MOST important to include in the first session?
A. An overview of the material that will be covered in all 3 sessions
B. A pretest to determine the level of knowledge of the participants
C. Active participation of the students in a low back evaluation
D. A complete demonstration of a low back evaluation

A

B An overview presented on the first day would imply that the therapist is not going to modify the series to the students’ current knowledge level, which may be ascertained from the pretest. It is most important for the physical therapist to know what level of knowledge the students have.
Although active participation is important for learning in every session, neither it, nor a complete demonstration of a low back evaluation, is most important to include in the first session.

126
Q

A patient with which of the following diagnoses would MOST likely benefit from pursed-lip breathing during exercise?
A. Peripheral vascular disease
B. Congestive heart failure
C. Emphysema
D. Sarcoidosis

A

C Peripheral vascular disease is a vascular problem, not a pulmonary problem. Congestive heart failure may lead to pulmonary problems, but not obstructive problems. Emphysema is an obstructive lung disease for which pursed lip breathing helps. Sarcoidosis is a restrictive lung disease for which pursed lip breathing does not help.

127
Q

A patient who had a complete spinal cord injury 1 week ago is referred to physical therapy for transfer training. The examination by the physical therapist reveals that the patient has Good (4/5) upper extremity strength, Zero (0/5) lower extremity strength, and fair sitting balance. Which of the following transfers is most appropriate to utilize INITIALLY with this patient?
A. Standing-pivot
B. Sliding-board
C. Sitting-pivot
D. 2-man dependent

A

B A standing-pivot transfer is a dependent transfer that should be used when the patient can bear some weight on the lower extremities. As a dependent transfer, it does not afford the patient the opportunity to progress towards independence as the sliding-board transfer would. A sliding-board transfer should be used when the patient has fair sitting balance, but does not have enough strength to fully lift all weight off the buttocks. A sitting-pivot transfer requires the patient to have enough upper extremity strength to fully lift the buttocks to clear the surfaces. It should be used as a progression from the sliding-board transfer, so it is not most appropriate to use initially. A 2-man dependent transfer is a dependent transfer that does not allow the patient to assist and is more than what is needed in this situation.

128
Q

A patient comes to physical therapy with a wound on the plantar aspect of the heel. The patient reports that the wound developed after stepping on a tack without realizing it. The wound is now round, measures 0.75 in (2 cm) in diameter, and displays no signs of infection. Which of the following initial interventions is MOST appropriate?
A. Bed rest
B. Custom molded shoe
C. Crutches with toe-touch weight-bearing
D. Total-contact casting

A

D Although bed rest would decrease pressures on the foot and may allow healing, it would also lead to deconditioning and other avoidable side effects. A custom molded shoe would not be sufficient as an initial treatment option, as the wound is still open. Crutches with toe-touch weight-bearing would help to decrease the pressure on the heal if the patient were able to appropriately follow instructions. However, as the patient most likely cannot feel the foot, there is a high probability of noncompliance. Also, it would not protect the foot from further injury as does the total-contact casting. Stepping on a tack without realizing it indicates that this patient has a neuropathy. The best way to treat a neuropathic foot ulcer is with total contact casting.

129
Q

A patient has pronounced weakness of the muscles of mastication on the left. A lesion in which of the following cranial nerves is MOST likely the cause of this impairment in the patient?
A. Trochlear (IV)
B. Trigeminal (V)
C. Facial (VII)
D. Hypoglossal (XIl)

A

B The trochlear nerve (IV) innervates the superior oblique muscle in the eye. The trigeminal nerve (V) innervates the masseter and temporalis, the major muscles of mastication. The facial nerve (VII) innervates the muscles of facial expression, not the muscles of mastication. The hypoglossal nerve (XII) innervates the tongue.

130
Q

A physical therapist is working with a patient in phase Il of a cardiac rehabilitation program. The patient’s current medications include a beta blocker and antidepressants. Prior to the start of exercise, the patient’s heart rate was 62 bpm, blood pressure - 110/75 mm Hg, and perceived rate of exertion - 6/20. After 10 minutes of aerobic exercise, the patient’s heart rate is 68 bpm, blood pressure - 120/70 mm Hg, and perceived rate of exertion - 14/20. Which of the following initial actions is MOST appropriate for the physical therapist to take now?
A. Continue the exercise at a lower intensity.
B. Continue the exercise at a higher intensity.
C. Continue the exercise at the same intensity.
D. Stop the exercise and contact the physician.

A

A Although the patient’s heart rate and blood pressure do not show abnormal changes, the perceived rate of exertion of 14 corresponds to “somewhat hard” to “hard”. This is too high for phase II cardiac rehabilitation, so the intensity should be decreased. The blunted response in the heart rate and blood pressure is due to the beta-blocker. Increasing intensity would mean working harder and would increase the patient’s perceived rate of exertion. As the perceived rate of exertion is already too high, this would be inappropriate. Because the perceived rate of exertion is too high, the same intensity should not be maintained. Although the perceived rate of exertion is high, it can be controlled by decreasing exercise intensity. It is presently unnecessary to contact the physician. If the patient abruptly stops exercise without a cool down, an increased arrhythmia may result.

131
Q

A patient is receiving physical therapy for a large wound on the lower leg. Today, after 3 weeks of treatment, the wound is free of necrotic tissue, but continues to have a copious amount of exudate. Which of the following dressings is MOST appropriate for the wound?
A. Saline-saturated gauze
B. Semipermeable film
C. Gauze impregnated with zinc oxide
D. Hydrocolloid paste

A

D
Saline-saturated gauze is not advised to control copious amounts of drainage.
Semipermeable film cannot absorb copious amounts of exudate. Gauze impregnated with zinc oxide has not been shown to be beneficial. Hydrocolloid paste is the only option given that is able to manage wounds with high levels of exudate.

132
Q

An athlete sustained a severe inversion sprain of the right ankle while playing basketball. To provide strapping support for the ankle, pressure and support should be applied over the tendons of the:
A. flexor digitorum longus and tibialis posterior
B. gastrocnemius and flexor hallucis longus
C. peroneus longus and peroneus brevis.
D. tibialis anterior and tibialis posterior.

A

C With an inversion injury, the tendons of the peroneus longs and brevis can become strained. Swelling with an inversion strain usually occurs over the anterolateral part of the ankle.
Compression and support is most beneficial by placing tape stirrups on the lateral side of the ankle, over the tendons of the peroneus longs and brevis and pulling the ankle into slight eversion

133
Q

A clean well-granulating Stage II pressure ulcer with minimal serous drainage is MOST appropriately dressed with:
A. a nonadherent dressing.
B. a packed dressing.
C. topical medication only.
D. dry-to-dry gauze.

A

A Anon-adherent dressing is best choice because any adherence of the dressing to the fragile granulating tissue could cause additional trauma. This is a superficial wound, so there would be no need to pack the wound. Since there is some drainage, a covering is preferential to topical medication only.

134
Q

A patient had final approval and checkout of a permanent prosthesis 2 weeks ago. The patient now reports to the physical therapist that the prosthesis is too heavy. The patient will benefit
MOST from:
A. an evaluation of the socket fit by the prosthetist.
B. a prescription from the physician for a lighter-weight prosthesis.
C. re-evaluation of hip strength by the physical therapist.
D. adding a fork strap attachment to the prosthesis.

A

C A complaint of heaviness likely reflects weakness. The complaint is not the fit or comfort of the prosthesis, so Option A is incorrect. The expense of obtaining a lighter prosthesis is not justified without determining if the solution is strengthening of muscle groups, which control the prosthesis. A fork strap attachment will assist with suspension, but will not reduce the weight of the prosthesis.

135
Q

A researcher who studied the effects of exercise training on balance test scores measured pre-exercise and post-exercise. Both the exercise group and the control group were randomly selected from a group of elderly adults residing in an assisted living facility. A t-test was used to compare the pre-test and post-test results. The balance scores were significantly improved in the group that performed regular exercises, but were unchanged in the control group. In deciding whether or not to incorporate these results into practice, a physical therapist who works in an adult outpatient clinic should be MOST concerned about which type of validity?
A. External
B. Internal
C. Construct
D. Statistical conclusion

A

A Threats to external validity limit the generalizability of the results. Since the research participants differ from the therapist’s patient population, applying the results on a different group of people would not be appropriate due to external validit. Internal validity means that internal factors other than the independent variables could be related to changes in the dependent variable(s). Construct validity is challenged when constructs are poorly defined or an inconsistent use of constructs occurs. Statistical conclusion validity is challenged with irregularities in the use of statistical tools or the use of incorrect statistical tools.

136
Q

A patient is asked to grip a white card between the thumb (1st digit) and index finger (2nd digit) with both hands. The physical therapist pulls on the card in the direction of the arrow shown. The starting and ending result of the movement is shown in the photograph. The results indicate weakness in which of the following muscles:
A. Flexor pollicis longus
B. Abductor pollicis brevis
C. Adductor pollicis
D. Extensor pollicis longus

A

C The action shown in the photograph is the Froment’s test. Both thumbs (1st digits) should stay extended during the test. If the thumb (1st digit) is flexed, it is indicative of weakness of the adductor pollicis with substitution by the flexor pollicis longs, which is usually due to a lesion of the ulnar nerve.

137
Q

A physical therapist is observing the gait of a patient with a transtibial prosthesis. The therapist observes that at heel strike (initial contact) the patient’s knee is hyperextended. What is the MOST likely cause of the patient’s gait deviation?
A. A heel cushion that is too soft
B. The socket is placed too far anterior to the foot
C. The prosthesis is too long
D. Inadequate suspension of the prosthesis

A

A A heel cushion that is too soft allows too rapid plantar flexion after heel strike (initial contact), which causes the knee to go into extension and stay there longer than normal. A socket that is placed too far anterior on the foot causes knee flexion, not extension. A prosthesis that is too long causes vaulting in midstance. Inadequate suspension causes the socket to slip during midswing phase resulting in the toe of the prosthesis catching on the floor.

138
Q

A patient presents to physical therapy with a 4-month history of low back pain. The patient has previously received therapy from other health care practitioners for this same pain, without resolution. During examination by the physical therapist, the patient reports a history of difficulty sleeping, concern that the pain will never improve, and irritability. The physical therapist should refer this patient to a:
A. social worker.
B. psychiatrist.
C. nutritionist.
D. physiatrist.

A

B Neither a social worker nor a nutritionist can adequately treat the patient’s depression. A patient exhibiting symptoms of chronic pain and depression warrants a referral to a psychiatrist or psychologist. A physiatrist can address the patient’s chronic pain, but is not as well equipped as a psychiatrist to address the patient’s depression.

139
Q

A patient who is 8-months pregnant has an abdominal diastasis recti with a separation of 1.5 in (4 cm). Which of the following exercises would be the MOST appropriate initial exercise for abdominal strengthening in a supine position?
A. Trunk curls
B. Hook-lying head lifts
C. Pelvic-tilt leg sliding
D. Bilateral leg lowering

A

B Trunk curls are contraindicated for a patient with diastasis recti. Supine hook-lying head lifts emphasize the rectus abdominus muscle and are least likely to increase the separation of the diastasis recti. Pelvic-tilt leg sliding is more advanced than head lifts. Bilateral leg-lowering is an advanced abdominal strengthening exercise that causes excessive low back strain and should not be performed during pregnancy.

140
Q

A physical therapist is obtaining the history of a patient, during an initial evaluation. The patient reveals a 5-year history of rheumatoid arthritis. The physical therapist should expect the patient to describe pain associated with the rheumatoid arthritis as:
A. morning pain with stiffness that improves with activity.
B. pain that is worse at the beginning of an activity.
C. constant pain that lasts throughout the day.
D. pain that increases throughout the day.

A

A Chronic inflammatory disorders such as rheumatoid arthritis are often associated with morning pain and stiffness that decreases throughout the day with activity. Pain that is worse at the beginning of an activity implies acute inflammation. Constant pain suggests tumors or visceral pain. Pain that increases throughout the day indicates increased congestion in a joint.

141
Q

A physical therapist is observing a patient from behind during bilateral shoulder abduction. The therapist notes that the patient’s right scapula is more abducted than the left scapula at the end range of movement. The MOST likely cause of the altered scapula position on the right is:
A. tightness of the rhomboid major and minor.
B. weakness of the serratus anterior.
C. restricted motion of the glenohumeral joint
D. weakness of the upper trapezius

A

C Tightness of the rhomboid major and minor would promote downward rotation of the scapula.
Weakness of the serratus anterior would limit the upward rotation of the scapula. The most likely reason for the increase in scapular motion is restriction of the glenohumeral joint. To fully abduct the shoulder, the scapula and glenohumeral joint both have to contribute to the motion. If the glenohumeral joint is restricted, the scapula has to increase its motion to accomplish the task.
Weakness of the upper trapezius would demonstrate a scapular lag in upward rotation.

142
Q

A physical therapist is evaluating a patient experiencing shoulder pain. The patient notices the shoulder pain when stocking shelves that are overhead at work. The pain is not apparent when stocking shelves at waist or chest level. The patient MOST likely has weakness in which of the following muscles?
A. Pectoralis minor
B. Upper trapezius
C. Deltoid
D. Rhomboid major

A

B Weakness in the pectoralis minor would not cause restriction of the scapula, but would likely cause scapular hypermobility. Weakness in the upper trapezius would decrease upward rotation of the scapula during shoulder flexion and abduction. The more the shoulder is elevated, the more noticeable this would be. The decreased scapular movement would increase the predisposition towards impingement. Weakness in the deltoid would cause the humerus to move downward, not upward, during shoulder elevation. Weakness in the rhomboid major would not cause restriction of the scapula, but would likely cause scapular hypermobility.

143
Q

A physical therapist is evaluating a patient with low back pain and associated symptoms. Which of the following findings should cause the therapist to refer the patient back to the physician?
A. Good (4/5) strength of the hamstrings
B. Pain radiating down the back of the thigh into the calf
C. Positive straight-leg test at 60° of hip flexion
D. Saddle anesthesia around the perineum

A

D Options A, B, and C are typical with low back pain and none alone requires immediate referral.
Saddle anesthesia around the perineum is a sign of a cauda equine syndrome and usually requires immediate treatment by someone other than a physical therapist.

144
Q

A physical therapist is planning to use electromyographic biofeedback to evaluate muscle activity in a patient who had a cerebrovascular accident. The therapist is monitoring activity in the patient’s triceps brachi muscle. Which method of electromyographic biofeedback signal processing would give the BEST measure of the quantity of muscle activity during a planned activity?
A. Integrated
B. Full-wave rectified
C. Amplified raw
D. Low-pass filtered

A

A
When using biofeedback there are several different options, which may be used to analyze the data. The best method for quantifying electromyographic biofeedback is to use an integrated form of the data. This process provides a summation of the data over a period of time. Full-wave rectified electromyographic biofeedback is performed prior to integration. Raw electromyographic biofeedback is not quantified without applying integration. Low-pass filtering electromyographic biofeedback is used to eliminate noise, but does not quantify the signal.

145
Q

On the first day following a patient’s total knee arthroplasty, a physical therapist establishes the use of a continuous passive motion device with a setting of 0° to 40° of motion. The MOST appropriate of the following reasons to use the continuous passive motion device is to:
A. decrease length of the patient’s hospital stay.
B. decrease prevalence of deep vein thrombosis in the patient.
C. regain knee flexion in the patient
D. prevent knee flexion contracture in the patient.

A

C There is no clear evidence that a continuous passive motion device reduces hospital stay or prevents deep vein thrombosis. Evidence suggests that a continuous passive motion device can help to regain knee flexion. If the knee is not placed in full extension outside the continuous passive motion device unit, a knee flexion contracture may result.

146
Q

A patient presents with ptosis of the left eye, a lateral strabismus, and a dilated pupil on the left.
The patient also reports double vision. Upon examination, results of which of the following cranial nerve tests is MOST likely to be abnormal?
A. Pupillary light reflex
B. Facial muscle strength
C. Jaw-jerk reflex
D. Pain sensation on the face

A

A The cranial nerve involved is the oculomotor nerve. This nerve innervates the medial rectus, which if weak, would cause a lateral strabismus. The oculomotor nerve is also responsible for mediating papillary constriction and a lesion would cause papillary dilation. The posis is caused by loss of innervation to the levator palpebrae superioris muscle, which elevates the eyelid. The double vision would be caused from the inability to move the eyeball normally since 4 of the 6 ocular muscles are controlled by the oculomotor nerve. The oculomotor nerve is also important in mediating the pupillary light reflex. The facial nerve innervates the muscles of facial expression.
The trigeminal nerve mediates the jaw-jerk reflex and pain sensation from the face.

147
Q

During steady rate exercise, the blood pressure in a normal person MOST typically responds with:
A. systolic pressure increase and no change in diastolic pressure
B. no change in systolic pressure and marked decrease in diastolic pressure.
C. systolic pressure decrease and diastolic pressure increase.
D. no change in systolic pressure, until the end point of the exercise bout.

A

A
Steady rate exercise such as jogging or cycling causes a relatively rapid increase in systolic pressure with diastolic pressure remaining the same or decreasing only slightly. Systolic pressure may level off somewhat during the exercise, if it remains at the same intensity.

148
Q

A patient fell while rock climbing 2 days ago. The fall resulted in a fracture of the right ankle, requiring an open reduction internal fixation. The patient also sustained a brachial plexus injury which resulted in significant weakness in the muscles in the right C6-C myotome. The patient is to remain non-weight-bearing on the right. Which of the following assistive devices is MOST appropriate for this patient?
A. Axillary crutches
B. Wheelchair
C. Standard walker
D. Forearm crutches

A

B The axillary crutches, standard walker, and forearm crutches all require adequate strength in the elbow extensors and latissimus dorsi, especially due to the non-weight-bearing status of the patient. The non-weight-bearing status on the right could indicate any of the options, however, because the patient has weakness of the muscles in the C6-C7 dermatome, the patient would have difficulty using any assistive device that requires use of the elbow extensors and latissimus dorsi muscles. Therefore, the best option is the wheelchair since it provides mobility.

149
Q

A physical therapist is taking the history of a patient with low back pain. Which of the following questions is BEST for the therapist to ask, if the therapist suspects the pain is caused by an inflammatory reaction?
A. “Is your pain constant or intermittent?”
B. “What activity bothers you the most?”
C. “Is it difficult to stand up straight after you’ve been sitting?”
D. “Does your pain radiate down into the leg?”

A

A All of the questions are important in history taking. However, constant pain is usually a hallmark of an inflammatory reaction. Mechanical pain generally changes with positions or activities. “What bothers you the most” is a good question to ask to find out what makes the pain better or worse and often helps determine the best course of intervention. Difficulty standing up from a sitting position is usually indicative of mechanical (disc) pain. Radiating pain could occur with mechanical or inflammatory disorders, so is not as discriminating as “Is your pain constant or intermittent?”

150
Q

Which of the following activities should be AVOIDED by a patient who has undergone a total hip arthroplasty using a posterolateral approach 5 days ago?
A. Walking with a walker
B. Isometric gluteal setting exercises
C. Active hip abduction in supine
D. Crossing the legs while seated in a chair

A

D Patients can begin walking 1 or 2 days postoperation with some weight-bearing or touch down depending on the surgical technique. They can also begin gluteal sets and active hip abduction from a supine position within 5 days. The posterolateral approach for a total hip arthroplasty requires postoperative precautions, which include no hip flexion beyond 80° or 90° and no hip adduction past the midline.

151
Q

A patient has a 2 week history of constant left, lower quadrant, abdominal pain that occasionally refers to the low back, pelvis, and left lower extremity. The patient also reports having a low grade fever and bloody, loose stools during the last few days. Which of the following conditions MOST likely explains the patient’s signs and symptoms?
A. Kidney stones
B. Diverticulitis
C. Gallstones
D. Appendicitis

A

B Kidney stones would most likely present with back pain and pelvic area pain and would not cause bloody stools. Diverticulitis is associated with constant, left, lower quadrant pain and tenderness along with bloody stools. The gall bladder is in the right upper quadrant. The appendix is in the right lower quadrant.

152
Q

During the gait evaluation of a patient, the physical therapist notices that the patient laterally bends excessively towards the right side during midstance phase on the right. In order to muscle test the suspected muscle for Normal strength (5/5) the patient should FIRST be positioned:
A. prone with the knee straight
B. supine with the knee bent.
C. seated with the hip flexed to 110°.
D. sidelying on the left side.

A

D Prone with the knee straight would be used to test for a Poor (2/5) grade. Supine with knee bent is used to isolate the gluteus maximus. Seated hip flexion is used to test the hip flexor muscle strength. The most likely cause of laterally bending towards the stance limb is abductor weakness on the stance side. The lateral bending helps compensate for weak abductors. The right abductors would be tested from a left sidelying position for fair grade (3/5) and above which is the correct starting position.

153
Q

During evaluation of a patient’s gait, the physical therapist observes that the patient leans forward shortly after heel strike (initial contact). The patient’s forward bending is MOST likely a compensation for weakness of which muscle(s)?
A. Quadriceps
B. Hamstrings
C. Gluteus maximus
D. Anterior tibialis

A

A The quadriceps are active shortly after heel strike (initial contact) to prevent excessive knee bending during the loading phase of initial stance. Weakness of the quadriceps causes the patient to compensate by leaning forward at heel strike (initial contact) and to use the body weight to help keep the knee extended. Weakness of the hamstrings causes excessive knee extension (recurvatum) prior to heel strike (initial contact). Weakness of the gluteus maximus would be observed shortly after heel strike (initial contact). However, the patient would lean backwards to compensate. Weakness of the anterior tibialis causes a “foot slap” just after heel strike (initial contact).

154
Q

A patient reports pain and weakness when performing overhead activities and has difficulty flexing the arm past 90° The physical therapist finds the results of the patient’s impingement tests are positive without a rotator cuff tear. Which of the following interventions should the therapist introduce FIRST in the patient’s rehabilitation program?
A. Closed kinetic chain strengthening of the deltoid muscle
B. Active assistive shoulder range of motion
C. Stretching the shoulder external rotator muscles
D. Strengthening of the subscapularis and teres major muscles

A

B
Later in the course of the patient’s treatment, strengthening exercises can be done, but first, the rotator cuff muscles, and then later, the deltoid. Early strengthening of the deltoid would worsen the impingement. This patient has an impingement syndrome. Generally, the first goal is to restore pain free range of motion. This can be done actively, passively or with the use of active assistive devices such as cane exercises. The external rotators should be strengthened, not stretched. External rotation would be emphasized which would mean stretching the internal rotators. The subscapularis and teres major muscles are internal rotators and should be stretched not strengthened. Strengthening them could potentially increase the impingement symptoms.

155
Q

A patient is being seen by a physical therapist following a mild myocardial infarction. The goal is to increase the patient’s exercise endurance for return to work. The patient is otherwise healthy and is taking propranolol (Inderal) 1 time/day. The therapist is setting the patient up on a daily home exercise program utilizing a stationary bike and treadmill walking. The therapist should teach the patient to use which of the following methods to monitor exercise intensity during exercise sessions at home?
A. Heart rate
B. Blood pressure
C. Perceived exertion
D. Respiration rate

A

C Normally, heart rate monitoring is the best method for monitoring exercise intensity.
However, since the heart rate is controlled, it should not be used. Blood pressure is a measure of intensity, but is not practical in this case. The effect of the propranolol (Inderal), a beta blocking agent, is to control heart rate and decrease the workload on the heart. Perceived exertion (Borg’s scale) is the preferred method to monitor exercise intensity for a patient taking beta blocking agents. Respiration rate may not be an accurate indicator of exercise intensity and cardiac workload.

156
Q

Which of the following stretching techniques is BEST for addressing an anteriorly tilting pelvis that is caused by muscle shortness?
A. Prone lying with the knee bent
B. Supine with the knee bent and pulled toward the chest
C. Quadruped sitting back on the heels with arms stretched forward
D. Lying on the back with hip flexed to 90° and knee straight

A

A
An anterior tilting pelvis is often caused by shortness in the hip flexors. The hip flexors can be stretched with a number of exercises that emphasize hip extension. Prone lying with the knees bent stretches both the iliopsoas and rectus femoris muscles. Supine with knees bent stretches the gluteus maximus. Quadruped with arms extended stretches the latissimus dorsi and low back, but does not address the iliopsoas, because the hips are fully flexed. Lying on the back with hip flexed and knee extended should be used to stretch the hamstrings.

157
Q

A physical therapist is examining the hip range of motion in a patient, as shown in the photograph. Passive range of motion is applied to the patient’s legs in the direction of the arrows.
The photograph shows the end points of the range of motion. The MOST likely cause of the hip dysfunction occurring in the patient is:
A. hypomobility of the hip internal rotators on the left.
B. weakness of the left hip internal rotators
C. laxity of the left hip capsule.
D. tightness of the hip external rotators on the left.

A

D Internal rotation of the left hip is limited. This could be caused by hypomobility of the left hip capsule or tightness of the left external rotators, especially the piriformis. Hypomobility of the internal rotators would cause limited external rotation.

158
Q

A physical therapist is treating a patient who has shoulder pain radiating down into the elbow.
The therapist applies transcutaneous electrical nerve stimulation, which gives initial relief, but then the pain returns. To prevent the nerve from accommodating to the stimulation, the therapist should FIRST:
A. decrease the intensity.
B. increase the pulse width.
C. switch to modulated mode.
D. change the electrodes.

A

C Decreasing the intensity reduces the effect. Increasing the pulse width increases the intensity of stimulation, but does not prevent adaptation. Normal nerves adapt to constant stimulation. Since the stimulation was effective initially, it may be assumed that the nerves were adapting to the stimulation. The purpose of modulation mode on a transcutaneous electrical nerve stimulation device is to prevent the adaptation. Therefore, this is the easiest and the most appropriate first action for the therapist to take. Changing the electrodes may be an appropriate action, but is not the appropriate first action

159
Q

A physical therapist is designing a rehabilitation program for a patient recently diagnosed with ankylosing spondylitis. The physical therapist should anticipate that as the disease progresses, the patient is MOST likely to require:
A. special precautions for osteoporosis.
B. a wheelchair for community mobility.
C. assisted ventilation.
D. bilateral ankle-foot orthoses.

A

A
Osteoporosis is a skeletal complication associated with longstanding ankylosing spondylitis.
Regarding requiring a wheelchair, the patient should still be able to walk even with advanced stages of ankylosing spondylitis. While lung expansion is generally decreased, assisted ventilation would not be required, because the muscles of respiration remain functional.
Peripheral neuropathies are not characteristic of ankylosing spondylitis

160
Q

A physical therapist is applying electrical stimulation to a patient with a neurapraxia. To minimize accommodation, the physical therapist should:
A. decrease the size of the stimulating electrode.
B. increase the pulse duration.
C. utilize a rapid rate of rise.
D. select a biphasic waveform.

A

C A decrease in size of the electrode will intensify the current density and is not a measure to minimize accommodation. An increase in the width of the stimulus increases the amount of time that the electrical stimulation is applied, but should not affect accommodation. A neurapraxia indicates that the muscle is innervated. Too slow a rise time results in changes in the tissue membrane known as accommodation, which gradually elevates the threshold required for the nerve to fire. Therefore, the rise time must be rapid enough to avoid accommodation. A biphasic waveform, also known as an alternating, bipolar, or faradic current, does not minimize accommodation.

161
Q

Sensory-level electrical stimulation is MOST appropriate for a patient with which of the following conditions?
A. Chronic low back pain of somatic origin
B. Acute ankle sprain with edema
C. Supraspinatus tendonitis
D. Active Raynaud’s syndrome

A

D Regarding chronic low back pain of somatic origin, the patient has a chronic problem, so will most likely require motor-level stimulation, because it provides a longer-lasting analgesia. There is no evidence to indicate that sensory-level stimulation is effective in the treatment of edema.
For edema reduction, rhythmic muscle contraction is preferred. lontophoresis is more appropriate than sensory-level electrical stimulation for treatment of a tendonitis, because it involves the delivery of anti-inflammatory medications. Raynaud’s syndrome is a condition in which the smallest arteries that bring blood to the fingers or toes constrict when exposed to cold or from an emotional upset. Sensory-level stimulation over nerve roots and trunks can increase peripheral vasodilatation.

162
Q

While working in a private practice clinic, a physical therapist observes a patient fall in the parking lot outside the office. The patient sustains a severe laceration to the forearm. The physical therapist secures a pressure dressing to the wound site, but notes that blood is soaking through the dressing and the bandage. Which of the following actions should the therapist perform NEXT?
A. Elevate the limb and apply pressure to the wound.
B. Remove the dressing and bandage and start over with a tighter bandage
C. Apply additional dressings and bandages and apply pressure to the brachial artery.
D. Call the patient’s physician and arrange transportation for medical care.

A

A Elevation and pressure is the correct next step in controlling bleeding. Removal of the dressing and bandage would potentially irritate the wound and increase the hemorrhage.
Elevation and pressure should be applied first, before additional dressings, bandages, and pressure to the brachial artery. While calling the patient’s physician to arrange transportation for medical care may be necessary, it does not immediately control the bleeding.

163
Q

A physical therapist is using a stethoscope to auscultate a patient’s S and Sa heart sounds. Of the following procedures, the MOST appropriate for the physical therapist to use is to apply the:
A. diaphragm of the head of the stethoscope firmlv on the skin.
B. bell of the head of the stethoscope firmly on the skin.
C. diaphragm of the head of the stethoscope lightly on the skin.
D. bell of the head of the stethoscope lightly on the skin.

A

A The diaphragm of the stethoscope is best for hearing higher-pitched sounds, such as S1 and S2. It is placed firmly on the skin to ensure good contact. The bell of the head of the stethoscope is recommended for S3, because the pitch is deeper, and is placed lightly on the skin, because increased pressure can cause the skin to act like a diaphragm preventing low-frequency sounds from being heard. Although the diaphragm of the head of the stethoscope is correct, light placement on the skin would not ensure adequate skin contact. Regarding option D, this technique would not ensure adequate skin contact. Also, the bell of the head of the stethoscope is recommended for S3 because the pitch is deeper.

164
Q

While performing transfer training from bed to chair with a patient who had a total knee arthroplasty 2 days ago, the electrocardiograph monitor alarms and the physical therapist notes that premature ventricular contractions have developed. What is the physical therapist’s BEST action at this time?
A. Position the patient on a stable surface and discontinue physical therapy for the day.
B. Continue the transfer to the chair and monitor the patient’s oxygen saturation levels.
C. Continue the patient’s transfer to the chair and immediately notify the nurse about the premature ventricular contractions
D. Position the patient on a stable surface and determine the stability of the premature ventricular contractions

A

D
Option A would apply only after the therapist has done further analysis of the situation.
Monitoring oxygen saturation is useful but not the most immediate thing to monitor. The therapist should monitor the electrocardiograph and blood pressure of the patient. While notifying the nurse is something the therapist should do, it should come after an initial evaluation of the stability of premature ventricular contractions. Onset of premature ventricular contractions can be either benign or stable. Less than 6 contractions/minute is generally stable, while greater than 6 contractions/minute is considered less stable. A physical therapist should be able to determine this stability. Premature ventricular contractions are entry level arrhythmias.

165
Q

A patient with chest pain from myocardial ischemia will MOST likely exhibit:
A. increased pain upon chest-wall palpation.
B. increased pain with deep breathing.
C. relief with nitroglycerin (Nitrostat) ingestion.
D. relief with antacid ingestion

A

C Increased pain with chest-wall palpation is more indicative of a musculoskeletal origin of pain. Increased pain with deep breathing is more indicative of a pulmonary origin of pain.
Nitroglycerin (Nitrostat) is a common vasodilator that is prescribed for patients who have angina.
Ingestion of a vasodilator will improve myocardial blood flow and help relieve ischemia and its manifestations. Relief of pain with antacid ingestion is more indicative of referred pain from peptic ulcer disease.

166
Q

Which of the following subjective reports from a patient with rheumatoid arthritis indicates the need for further medical examination by a physician?
A. Morning pain in both lower extremities
B. Numbness in both lower extremities
C. Increased pain during activities of daily living
D. Inability to sleep at night

A

B Morning pain is a typical finding in rheumatoid arthritis. Symmetrical numbness could be indicative of myelopathy from either infectious or neoplastic causes. Increased pain during activities of daily living is also a typical finding in rheumatoid arthritis. Inability to sleep at night alone is too vague of a report to cause suspicion of more medical problems.

167
Q

To acquire specific information about the visual status of a patient following an acute cerebrovascular accident, the physical therapist should refer to what section in a physician’s admission note?
A. PERRLA
B. CBC
C. Cor
D. PMH

A

A PERRLA stands for pupils, equal, round and reactive to light and accommodation; and is where physicians would document visual information. CBC stands for complete blood count and this section should not contain information about vision. Cor stands for heart and should not contain information about vision. PH stands for past medical history and is not necessarily specific about visual information.

168
Q

A patient with Parkinson’s disease demonstrates shortness of breath with activity. Which of the following tests is MOST appropriate for the physical therapist to perform to help delineate a cause for the patient’s shortness of breath?
A. Deep tendon reflex testing
B. Sensory examination
C. Muscle strength testing
D. Posture examination

A

D Changes in reflexes would hinder balance and equilibrium, more so than ventilation. Changes in sensation are less pronounced in Parkinson’s as compared to motor changes. Depending on a patient’s level of tone, muscle strength is difficult to accurately examine in patients with Parkinson’s and also would be less likely to directly contribute to shortness of breath. With regard to posture examination, as bradykinesia and rigidity evolve in Parkinson’s disease, concomitant kyphosis also develops. Thoracic kyphosis contributes to a restriction in ventilation and subsequent shortness of breath.

169
Q

During an initial evaluation, which of the following tests is MOST appropriate to perform with a patient who has acute right-sided congestive heart failure?
A. Sensory testing of upper extremities
B. Pitting edema measurements in the lower extremities
C. Resisted manual muscle testing of all extremities
D. Reflex testing of lower extremities

A

B While sensory testing is important in an initial exam, impaired sensation is less likely to occur in congestive heart failure by itself (as written in the question). Right-sided congestive heart failure results in dependent edema; measurements of pitting edema are appropriate to determine the severity of congestive heart failure and aid the therapist in treatment planning. Manual muscle testing is also appropriate for an initial exam. However, with acute congestive heart failure, resisted manual muscle testing is generally avoided until the congestive heart failure is more stable. Reflex testing is also important in an initial exam, but again, since this patient only has congestive heart failure, then reflexes are less likely to be impaired.

170
Q

A physical therapist is working on progressive functional mobility with a patient who had a transverse colectomy 2 days ago. The patient has developed a low grade fever. What is the MOST appropriate examination for the physical therapist to perform, prior to continuation of the patient’s intervention?
A. Heart rate
B. Blood pressure
C. Respiratory rate
D. Auscultation

A

D Changes in heart rate, blood pressure, or respiratory rate can accompany fever but none will help in delineating the cause of the fever. Common causes of immediate postoperative fevers are atelectasis or pneumonia. Auscultation of the lungs would help the therapist in delineating this as a cause of the fever.

171
Q

Upon auscultation, a physical therapist hears crackles over the third intercostal space and right midclavicular line. To MOST appropriately document this finding, what region of the lung should be identified by the therapist?
A. Anterior segment of the right upper lobe
B. Superior segment of the right middle lobe
C. Apical segment of the right upper lobe
D. Inferior segment of the right middle lobe

A

A The surface anatomy of the third intercostal space, right midclavicular line corresponds to the anterior segment of the right upper lobe. The surface anatomy of the third intercostal space, right midclavicular line does not correspond to the right middle lobe, right upper lobe, or right middle lobe.

172
Q

In addition to standard precautions, what other precaution should a physical therapist observe when working with a patient infected with methicillin-resistant Staphylococcus aureus?
A. Airborne
B. Sterile
C. Droplet
D. Contact

A

D Since methicillin-resistant staphylococcus aureus is spread by contact, wearing a face shield or mask is not necessary. Sterile precautions or techniques are not necessary for the physical therapist to use with methicillin-resistant staphylococcus aureus. The mode of transmission of methicillin-resistant staphylococcus aureus is contact.

173
Q

A 50 year-old patient with metabolic (insulin resistance) syndrome comes to physical therapy to initiate a conditioning program. Which of the following initial exercise prescriptions is MOST appropriate for this patient?
A. Intensity: Heart rate - 74 to 80 bpm. Frequency: 4 to 5 days/week. Duration: 30 minutes. Mode:
walking
B. Intensity: Heart rate - 105 to 115 bpm. Frequency: 2 to 3 days/week. Duration: 20 minutes.
Mode: walking.
C. Intensity: Rate of Perceived Exertion - 6 to 8 (6 to 20 scale). Frequency: 4 to 5 days/week.
Duration: 30 minutes. Mode: stationary bike.
D. Intensity: Rate of Perceived Exertion - 13 to 15 (6 to 20 scale). Frequency: 2 to 3 days/week.
Duration: 20 minutes. Mode: stationary bike.

A

A Option A’s exercise prescription is most ideal to begin an exercise program, because it satisfies the criteria of increasing energy expenditure while promoting cardiovascular adaptations and weight loss. The key feature in this prescription is the exercise intensity, as it is on the lower range of a target heart rate range (50% of age-adjusted maximum) that would allow accommodation to this program. The intensity in option B’s prescription is too high (greater than 75% of the age adjusted maximum) to begin with and could be detrimental to exercise adherence.
The rate of perceived exertion level in option C’s prescription is too low and would not achieve sufficient energy expenditure and exercise adaptation. The rate of perceived exertion level in this option D’s prescription is too high and could be detrimental to promoting exercise adherence.

174
Q

A physical therapist is performing sit-to-stand transfer training with a patient in a hospital room.
The patient is currently admitted for acute renal failure and has electrocardiogram monitoring in place. During the transfer training, the therapist notes new onset of 1 unsustained, unifocal premature ventricular contraction. Which of the following actions is MOST appropriate for the therapist to take at this time?
A. Discontinue transfer training and call the nurse immediately.
B. Discontinue transfer training and switch to passive range of motion exercises
C. Allow the patient to rest and continue with transfer training, while monitoring the electrocardiogram.
D. Allow the patient to rest and measure the patient’s blood pressure.

A

C An unsustained unifocal premature ventricular contraction is a stable electrocardiograph change associated with activity and therefore doesn’t need an modification of the current intervention. Since this type of premature ventricular contraction is stable and there is no report of dizziness in the stem, then taking blood pressure is not necessary at this time.

175
Q

A patient had a split-thickness skin graft for a superficial partial-thickness burn injury to the upper extremity. The surgeon has requested range of motion exercises for the patient. Currently, the patient is able to actively move the upper extremity through one-third of range of motion for shoulder flexion. Based on this finding, what is the MOST appropriate action for the physical therapist to take at this time?
A. Defer any range of motion exercises until the patient is able to participate more actively.
B. Begin active assistive range-of-motion exercises.
C. Begin bed mobility training to facilitate increased use of the upper extremity.
D. Continue with active range-of-motion exercises.

A

B Deferring any range of motion is not a practical choice, as contracture postoperatively will develop. Since this patient cannot achieve full range of motion by himself, then active assistive range of motion is indicated to prevent contracture postoperatively. While bed mobility training is a creative way to possibly increase upper extremity range of motion, given the acuity of the patient’s surgical wound, the patient would need more range of motion for this intervention to be more beneficial. Continuing with only active range of motion would not facilitate adequate increases in range of motion and would not prevent contractures.

176
Q

A patient with idiopathic pulmonary fibrosis completed a 6-minute walk test and demonstrates the following results: Total Walking Distance - 1200 ft (366 m) in 6 minutes; Heart Rate - 82 to 110 bpm (pre-test to post-test); Blood Pressure - 125/80 to 145/85 mm Hg (pre-test to post-test);
Respiratory Rate - 18 to 40 (pre-test to post-test); Oxygen Saturation - 98% to 92% (pre-test to post-test); Electrocardiogram - Normal sinus rhythm throughout test. Based on these results, the physical therapist should determine that this patient has impaired:
A. aerobic capacity and endurance associated with cardiovascular pump dysfunction.
B. ventilation, respiration, and aerobic capacity associated with airway clearance dysfunction.
C. ventilation, respiration, aerobic capacity and gas exchange associated with ventilatory pump dysfunction.
D. aerobic capacity and endurance associated with cardiovascular pump failure.

A

C Based on the walk test results the heart rate and blood pressure have normal physiologic rise to exercise and would not indicate cardiovascular pump dysfunction. While the walk test results do indicate impaired ventilation and respiration, there is no indication of airway clearance issues in the question. In general a patient with pulmonary fibrosis will have an impaired ventilatory pump. This is further evidenced by the exaggerated respiratory rate response and desaturation in the 6-minute walk test results

177
Q

A patient who is currently participating in an outpatient cardiac rehabilitation program asks the physical therapist about intervention options for an apparent athlete’s foot fungal infection on both of the patient’s feet. The patient has been using an over-the-counter analgesic ointment for 5 days, but the condition does not seem to be improving. Which of the following actions is the correct response of the physical therapist?
A. Refer the patient back to the cardiologist.
B. Recommend an over-the-counter anti-fungal ointment.
C. Tell the patient to continue using the ointment for another 5 days and then reassess.
D. Instruct the patient to discuss the situation with a pharmacist.

A

D
The cardiologist is not the best referral, as this appears to be an integumentary problem unrelated to the cardiac condition. A physical therapist should not recommend any over the counter medications, as it is outside the scope of practice of a physical therapist. Any pharmaceutical education or advice, except “as prescribed by your physician”, is outside the physical therapist’s scope of practice. A pharmacist is the most appropriate heath care professional of those given to assist this patient.

178
Q

A physical therapist is reviewing the medical record of a patient in the intensive care unit. The patient was admitted the previous night through the emergency room, following a motorcycle accident resulting in a fractured right femur. The therapist notes a physician’s order for a Doppler study of the left leg. The therapist should:
A. proceed with the evaluation and intervention without any changes.
B. hold physical therapy, until results of the study are obtained and interpreted by the physician
C. proceed with the evaluation and limit intervention to transfer to a bedside chair.
D. obtain clearance from the nurse to provide intervention for the patient

A

B A complete physical therapy evaluation and treatment is contraindicated due to a possible deep vein thrombosis. A physician’s order for a Doppler study indicates possible deep vein thrombosis, so the physical therapy should not be conducted until the Doppler study is completed and the results analyzed by the physician. Transfer from bed to chair contraindicated due to possible deep vein thrombosis. The nurse alone should not be providing clearance, until the Doppler study has been completed and interpreted.

179
Q

A physical therapist is conducting a reflex text as shown in the photograph. The starting and ending position of the test is indicated by the arrow. The results of the test are MOST likely to indicate:
A. a peripheral nerve injury.
B. a lesion of the anterior horn cells.
C. a normal response.
D. an injury to the spinal cord.

A

D The therapist is testing for a positive Babinski’s sign. When the Babinski’s sign is positive, the toes flare at the end of the test. The presence of a positive Babinski’s sign indicates an upper motor neuron lesion such as a spinal cord injury. If the patient is normal, the toes will flex. The only upper motor neuron lesion is injury to the spinal cord. Peripheral nerve injuries and lesions of the anterior horn cells are lower motor neuron lesions.

180
Q

A 3 month-old infant presents to physical therapy with poor midline head control. Upon evaluation, the physical therapist notes facial asymmetry and observes limitation of cervical rotation to the left and cervical lateral flexion to the right in the infant. A radiology report indicates premature fusion of the infant’s cranial sutures. This infant has:
A. right congenital muscular torticollis.
B. left congenital muscular torticollis.
C. right cervical facet hypomobility.
D. left cervical facet hypomobility.

A

A
The infant exhibits signs of torticollis affecting right sternocleidomastoid muscle. Torticollis is named for the side of the limited lateral flexion. Asymmetry and premature closure of sutures (plagiocephaly) is not typically seen with cervical facet hypomobility in infants.

181
Q

A physical therapist evaluates a patient who has back pain and determines that the patient’s pes plans is contributing to this pain. Which of the following orthotic interventions is MOST appropriate for this patient?
A. Metatarsal pad
B. Solid ankle-foot orthosis
C. Hinged ankle-foot orthosis
D. Longitudinal arch support

A

D Neither a metatarsal pad, a solid ankle-foot orthosis, nor a hinged ankle-foot orthosis corrects a longitudinal arch. The longitudinal arch support is the only orthotic given that will address pes planus.

182
Q

During which of the following scenarios are gloves required to comply with standard precautions?
A. During all patient care in the hospital setting
B. Performing range of motion on a patient with acquired immune deficiency syndrome
C. Massaging the neck of a patient with hepatitis C
D. Changing an infant’s diaper in the pediatric setting

A

D Gloves are only required in contact with blood or body fluids, not during patient care that does not involve coming in contact with blood or body fluids. Gloves are required whenever changing an infant’s diapers, since this involves coming into contact with body fluids.

183
Q

A physical therapist is educating a patient on the use of a moist hot pack for home treatment. To prevent burns and still receive the benefits of superficial heat, which of the following heat application time frames is MOST appropriate for the therapist to recommend to the patient?
A. 5 to 10 minutes
B. 20 to 30 minutes
C. 45 to 60 minutes
D. 61 to 90 minutes

A

B Five to 10 minutes is an insufficient amount of time for therapeutic heating effects. The ideal amount of time for therapeutic heating effects with minimal risk of burns is 20 to 30 minutes.
Forty-five to 60 minutes is to long a period of time, as there is an increased risk of burn. Sixty-one to 90 minutes is also too long and has a significantly increased risk of burn

184
Q

A patient comes to physical therapy via direct access for evaluation of insidious shoulder pain.
Upon inspection, the physical therapist notes a yellowish color of the patient’s sclera and skin.
The therapist should refer this patient to a physician for probable primary dysfunction of the:
A. liver.
B. eye.
C. duodenum.
D. heart.

A

A The patient is exhibiting jaundice consistent with liver or gallbladder disorder. Eye involvement (sclera) is a symptom, not the primary disorder. There are no indications of duodenal involvement or cardiac involvement in this patient

185
Q

A 22 year-old patient is hospitalized awaiting a lung transplant due to cystic fibrosis. The patient’s physician is interested in an objective measure of the patient’s preoperative endurance.
Which of the following tests is MOST appropriate for the physical therapist to administer to this patient?
A. VO2 max treadmill test
B. 2-step test
C. Submaximal exercise test on a cycle ergometer
D. 6-minute walk test

A

D The patient is too ill for either the VO2 max treadmill test or the 2-step test. The submaximal exercise test is not as sensitive or specific a test as the 6-minute walk test. The 6-minute walk test is designed for acutely ill cardiopulmonary patients.

186
Q

A physical therapist is preparing for gait training with a young, adult patient with paraplegia.
Which of the following gait training options is MOST appropriate for the patient’s first session?
A. Swing-through gait pattern with a walker
B. Swing-through gait pattern with forearm crutches
C. Swing-to gait pattern with axillary crutches
D. Swing-to gait pattern in the parallel bars

A

D Option A is an inappropriate assistive device for a young adult with paraplegia. Option B is the ultimate goal of gait training, but too difficult for the patient’s first attempt. Option C is not the most efficient gait aide for a patient with paraplegia. The first time session of gait training for a patient with paraplegia should be in the parallel bars. A swing-to gait pattern would be the easiest for this patient to learn initially

187
Q

A patient with peripheral vascular disease presents to physical therapy for evaluation and intervention. The patient used to walk for exercise, but can no longer walk to the mailbox at the end of the driveway without experiencing leg pain. This patient will MOST likely also have:
A. relief of pain with the legs elevated.
B. purple or brown pigmentation of the skin on the legs.
C. relief of pain with the legs in the dependent position.
D. a positive Homan’s sign.

A

C Elevating the legs in the presence of arterial insufficiency decreases blood flow, which increases pain. Purple or brown pigmentation of the skin on the legs is associated with venous insufficiency, not arterial insufficiency. The patient has intermittent claudication caused by arterial insufficiency. Placing the patient’s legs in the dependent position facilitates blood flow and reduces pain. Pain with exercise indicative of intermittent claudication, not deep vein thrombosis associated with a positive Homan’s sign.

188
Q

A patient comes to physical therapy for intervention for an ulcer on the right medial malleolus.
Upon inspection of the patient’s legs, the physical therapist notes normal skin temperature and brownish pigmentation of the skin. The patient MOST likely also has:
A. edema.
B. diminished pedal pulse.
C. lower extremity pain with exercise.
D. hypersensitivity to cold.

A

A
Skin temperature, color, and presence of ulcer are consistent with chronic venous insufficiency. Options B and C are characteristic of arterial insufficiency, not venous insufficiency. Option D is characteristic of Raynaud’s, not venous insufficiency.

189
Q

The parent of a 4 year-old child who has myelomeningocele is interested in obtaining orthoses for the child’s gait training. The child has an L1 neurological level lesion. Which of the following orthoses is the MOST appropriate selection for the physical therapist to discuss with the parent?
A. Hip-knee-ankle-foot with locked hips
B. Reciprocating-gait
C. Knee-ankle-foot with a pelvic band
D. Ankle-foot

A

B Options A, C, and D would provide insufficient orthotic support for the child to walk. A child with L1 lesion can only walk with support of reciprocating gait orthoses or thoracic lumbar sacral orthosis.

190
Q

A patient who sustained a right cerebrovascular accident presents with a flaccid arm. During muscle testing, the patient is able to shrug the left shoulder. The MOST accurate explanation for shoulder movement is that the right cerebrovascular accident:
A. has affected the right shoulder and not the left shoulder.
B. did not affect the vagus nerve (X), which innervates the upper trapezius muscle.
C. did not affect spinal accessory nerve (XI), which innervates the upper trapezius muscle
D. has affected the left biceps and triceps muscles, but not the deltoid muscles.

A

C A right cerebrovascular accident affects the left shoulder, not the right shoulder. The upper trapezius is controlled by spinal accessory nerve (XI), not the vagus nerve (X). The spinal accessory nerve (XI) (supplied by the corticobulbar tract) was apparently not affected by the stroke and accounts for the patient’s ability to shrug a flaccid arm. The deltoid does not shrug the shoulder

191
Q

A physical therapist is evaluating the cranial nerves of a child who has a medulloblastoma. The child’s right eye deviates medially. This child has impairment of which of the following cranial
nerves?
A. Oculomotor (Ill
B. Trochlear (IV)
C. Abducens (VI)
D. Vagus (X)

A

C The oculomotor nerve (III) controls the inferomedial eye muscles. The trochlear nerve (IV)
controls inferolateral eye movement. The abducens nerve (VI) controls lateral eve movement.
Damage to this nerve causes the eyeball to deviate medially due to weakness of the lateral rectus muscle. The vagus nerve (X) does not control the eye muscles.

192
Q

A physical therapist is examining a 9 month-old child who was born at 28 weeks gestational age.
Which of the following activities should this child be able to perform?
A. Walking independently
B. Standing independently
C. Creeping on hands and knees
D. Rolling supine to prone

A

D Walking independently is not achieved until 12-months adjusted age. Standing independently is not achieved until 11-to-12-months adjusted age. Creeping on hands and knees is not achieved until 9-months adjusted age. As the child is only 6 months old (adjusted age) developmentally, the child should only be able to roll.

193
Q

A 90 year-old hospitalized patient was referred for physical therapy evaluation and intervention following a C1 fracture secondary to a fall. The patient describes neck pain and left knee pain, but reports no other postinjury changes in the extremities. The rehabilitation prognosis for this patient should PRIMARILY be based on:
A. prior level of function.
B. left-knee range of motion.
C. use of a cervical collar.
D. upper-extremity sensory integrity.

A

A Understanding a patient’s prior level of function is the most critical factor in a patient’s history to determine a reasonable prognosis, as this establishes the baseline for recovery.
Although knee range of motion and upper-extremity sensory integrity are impairments that should be monitored, neither is more critical than prior level of function as a determinant of prognosis. Although cervical collar is a potential intervention, it is not more critical than prior level of function as a determinant of prognosis.

194
Q

A patient with a traumatic brain injury is being discharged to home following completion of inpatient rehabilitation. Which of the following assessment tools would BEST assess the patient’s potential?
A. Glasgow Outcome Scale
B. Fugl-Mever Assessment
C. Rancho Los Amigos Levels of Cognitive Functioning Scale
D. Sickness Impact Profile

A

C The Glasgow Outcome Scale is not as descriptive as the Rancho Los Amigos Levels of
Cognitive Functioning Scale and is used more often as a categorization of traumatic brain injury states for research purposes. The Fugl-Meyer Assessment should be used only with patients who have had a stroke. The Rancho Los Amigos Levels of Cognitive Functioning is a descriptive scale that outlines a predictable sequence of cognitive and behavioral recovery in patients with traumatic brain injury. The Sickness Impact Profile is used to measure general health status and is self-administered; therefore, it would not be useful as a prognostic discharge tool for a patient with a traumatic brain injury.

195
Q

A patient with chronic obstructive pulmonary disease is starting to participate in a supervised exercise program. The MOST important aspect to monitor during this exercise program is the:
A. cardiorespiratory endurance training.
B. resistive exercise.
C. active soft-tissue stretching.
D. warm-up and cool-down intensity.

A

D
Cardiorespiratory endurance training should occur following the warm-up period and should be monitored; however, the monitoring should have occurred prior to this time. Resistive exercises occur following the cool-down period and therefore monitoring should have occurred prior to this time. There are minimal needs for monitoring the active stretching. Cardiovascular complications during an exercise session are most likely to occur during warm-up and cool-down periods.

196
Q

During gait evaluation, a physical therapist notes that a patient demonstrates a shorter step length with the right lower extremity. Which of the following problems is MOST likely the cause of this gait dysfunction?
A. Right iliopsoas contracture
B. Painful left knee
C. Decreased ankle pronation on the right
D. Left gluteus medius weakness

A

B Rightiliopsoas contracture may cause a shorter step length with the left lower extremity, but the not the right lower extremity. Left knee pain will cause the patient to spend less time in left-sided stance as the patient will try to minimize the time spent in stance weight-bearing on the knee) to minimize the pain. Therefore, the patient will take a shorter step with the right lower extremity. Decreased ankle pronation would not have an effect on right-sided step length.
Gluteus medius weakness would be seen as an increase in lateral pelvic tilt, not step length

197
Q

A patient is receiving mechanical intermittent cervical traction with an on/off duty cycle of 20 sec/10 sec. The patient reports increased pain each time the traction unit cycles on, which then subsides over the duration of the on time. Which of the following modifications to the duty cycle is MOST appropriate?
A. Increase the off time to 20 seconds.
B. Increase the on time to 20 seconds.
C. Decrease the off time to 5 seconds.
D. Decrease the on time to 5 seconds.

A

A
When patients have severe symptoms, it is often useful to have increased on and off times to reduce the movement caused by the cycling. Increasing the off time allows for longer recovery time prior to re-initiating the traction and reduces the cycling. As the pain symptoms decrease, the relaxation time can be decreased. Options B, C, and D do not allow for a longer off or recovery period.

198
Q

A patient, who had a Cs traumatic spinal cord injury 1 month ago, reports a severe, pounding headache and restlessness while standing in the parallel bars. The patient’s vital signs indicate hypertension and bradycardia. The MOST appropriate immediate response of the physical therapist is to:
A. instruct the patient in deep breathing techniques.
B. apply a cervical cold pack.
C. remove the abdominal gait belt.
D. assess for catheter blockage.

A

D Neither deep breathing techniques, nor a cervical could pack, would be an effective response to this emergency situation. Although the abdominal gait belt might be a contributing factor, it is not the most likely primary cause. This patient scenario is indicative of autonomic dysreflexia, a pathological reflex. The most common cause is bladder distention or urinary retention, which is likely to be caused by a urinary catheter blockage.

199
Q

A physical therapist is testing the deep tendon reflex of a patient, as shown in the photograph.
The patient has a partial nerve injury to the tested nerve root. Which of the following reflex grades is the therapist MOST likely to find in the patient?
A. 0
B. 1 +
C. 2+
D. 3 +

A

B A grade of 0 indicates complete severance of the nerve. A partial nerve injury causes a diminished reflex grade, which is graded as 1+. A 2+ grading is normal. A 3+ grade indicates an upper motor neuron lesion, not a peripheral nerve injury.

200
Q

During an initial physical therapy evaluation, a patient reports occasional breathlessness. Which of the following patient reports indicates the PRIMARY need for a more thorough pulmonary examination by the physical therapist?
A. There is paroxysmal nocturnal dyspnea.
B. The symptoms are relieved by leaning forward.
C. There has been a recent change in physical activities.
D. The breathlessness is associated with lightheadedness.

A

B
Paroxysmal nocturnal dyspnea is common in severe heart disease. Relieving the breathlessness symptoms by a positional change is suggestive that the problem is pulmonary in origin. Occasional breathlessness may be expected with a change in physical activities, but more history should be taken to expand. Lightheadedness is more indicative of a cardiac problem.